Sie sind auf Seite 1von 58

30.

Surgery- General Surgery

1
A 3 year old child presents to the emergency room with a 2 day history of painless rectal bleeding.

2
Massive painless rectal bleeding is most likely to be seen in children with which of the following conditions:

duodenal atresia/intussusception
pyloric stenosis/midgut volvulus
lactose intolerance/annular pancreas
Prune-Belly (Eagle-Barrett) syndrome
Meckel's diverticulum/intestinal duplication

Explanation

The correct answer is choice E.

Meckel’s diverticulum is the most common congenital abnormality of the small intestine; it is caused by an
incomplete obliteration of the vitelline duct. In most cases, it contains ectopic gastric tissue that secretes
gastric acid leading to ulceration of the surrounding tissues, Bleeding from these ulcer usually ends into
painless bleeding per rectum.

Alimentary tract duplication is rare developmental error that has been assigned several different names,
including enterocystomas, enterogenous cysts, supernumerary accessory organs, ileum duplex, giant
diverticula, and unusual Meckel’s diverticulum. In this condition abrupt hemorrhage can be encountered in
the case of a cyst lined with gastric mucosa that ulcerates and eventually erodes into adjacent organs
and/or vessels.

Answer A is false: In duodenal atresia there is no bleeding per rectum and even there is no passage of
meconium. In intussusception there is painful colicky passage of stools that look like currant jelly. This is a
mixture of mucus, sloughed mucosa, and shed blood.

Answer B is false: In pyloric stenosis there is nonbilious vomiting. There may be Slight hematemesis of
either bright red flecks or a coffee-ground appearance but no rectal bleeding occurs.

Midgut vovulus is presented by abdominal distension, visible peristaltic in the epigastrium, bilious or
nonbilious emesis and bloody diarrhea associated with severe abdominal pain.

Answer C is false: Annular pancreas is similar to duodenal atresia and not presented by bleeding per
rectum and even there is no passage of meconium. Lactose intolerance is presented by bloating, abdominal
discomfort, meteorism, flatulence and passage of loose, watery and acidic stool that occur from 1 hour to a
few hours after ingestion of milk or dairy products associated with functional recurrent abdominal pain.

Answer D is false: Prune belly syndrome typically occurs in boys with a thin or lax abdominal wall. It is
characterized by long prostatic urethra, prostatic hypoplasia, tortuous and dilated ureters, hydronephrosis
and varying degrees of renal dysplasia. It may be associated wit GI anomalies as malrotation with
mesenteric defect, imperforate anus, gastroschisis, Hirschsprung disease, and constipation.

An asymptomatic 66 year-old man has a routine complete blood count which reveals a leukocytosis of
50,000 white blood cells per microliter. On fluorescent in situ hybridization assay, he is found to have the
Philadelphia chromosome in peripheral blood mononuclear cells, and he is diagnosed with chronic
myelogenous leukemia. He subsequently undergoes radiation therapy and a bone marrow transplantation.
Within thirty days, he develops a maculopapular rash, abdominal pain, and severe nausea and vomiting.

Which one of the following is the cause of the patient’s symptoms?

3
Complement
Preformed antibodies
Cytokines
NK cells
Macrophages

Explanation

The correct answer is choice C

Graft-versus-host disease (GVHD) is a common complication of allogeneic bone marrow transplantation in


which functional immune cells in the transplanted marrow recognize the recipient as "foreign" and mount
an immunologic attack. GVHD is an inflammatory disease that is unique to allogeneic transplantation. This
can occur even if the donor and recipient are HLA-identical because the immune system can still recognize
other differences between their tissues. It is aptly named graft-versus-host disease because bone marrow
transplantation is the only transplant procedure in which the transplanted cells must accept the body
rather than the body accepting the new cells.

Clinically, graft-versus-host-disease is divided into acute and chronic forms.

 The acute or fulminant form of the disease (aGVHD) is normally observed within the first 100 days
post-transplant and is a major challenge to transplants owing to associated morbidity and
mortality. Acute graft-versus-host disease typically occurs in the first 3 months after
transplantation and may involve the skin, intestine, or the liver, and is often fatal.
 The chronic form of graft-versus-host-disease (cGVHD) normally occurs after 100 days. The
appearance of moderate to severe cases of cGVHD adversely influences long-term survival.

Classically, acute graft-versus-host-disease is characterized by selective damage to the liver, skin and
mucosa, and the gastrointestinal tract. Chronic graft-versus-host-disease also attacks the above organs,
but over its long-term course can also cause damage to the connective tissue and exocrine glands. Acute
GVHD of the GI tract can result in severe intestinal inflammation, sloughing of the mucosal membrane,
severe diarrhea, abdominal pain, nausea, and vomiting. This is typically diagnosed via intestinal biopsy.
Liver GVHD is measured by the bilirubin level in acute patients. Skin GVHD results in a diffuse
maculopapular rash, sometimes in a lacy pattern.

After bone marrow transplantation, T cells present in the graft, either as contaminants or intentionally
introduced into the host, attack the tissues of the transplant recipient after perceiving host tissues as
antigenically foreign. The T cells produce an excess of cytokines, including TNF-α and interferon-gamma
(IFNγ). A wide range of host antigens can initiate graft-versus-host-disease, among them the human
leukocyte antigens (HLAs). However, graft-versus-host disease can occur even when HLA-identical siblings
are the donors. HLA-identical siblings or HLA-identical unrelated donors often have genetically different
proteins (called minor histocompatibility antigens) that can be presented by MHC molecules to the
recipient's T-cells, which see these antigens as foreign and so mount an immune response.

High-dose corticosteroids such as prednisone are a standard treatment; however this immuno-suppressive
treatment often leads to deadly infections. Chronic graft-versus-host disease may also develop after
allogeneic transplant. It is the major source of late treatment-related complications, although it less often
results in death. In addition to inflammation, chronic graft-versus-host disease may lead to the
development of fibrosis, or scar tissue, similar to scleroderma; it may cause functional disability and
require prolonged immunosuppressive therapy.

Complement (choice A) consists of a number of small proteins found in the blood, generally synthesized by
the liver, and normally circulating as inactive precursors. When stimulated by one of several triggers,
proteases in the system cleave specific proteins to release cytokines and initiate an amplifying cascade of

4
further cleavages. The end-result of this activation cascade is massive amplification of the response and
activation of the cell-killing membrane attack complex. Over 25 proteins and protein fragments make up
the complement system, including serum proteins, serosal proteins, and cell membrane receptors. They
account for about 5% of the globulin fraction of blood serum. Graft-versus-host-disease is not mediated by
the complement system

Preformed antibodies (choice B) causes hyperacute rejection in transplanted tissues. It is a complement-


mediated response in recipients with pre-existing antibodies to the donor (for example, ABO blood type
antibodies). Preformed antibodies does not cause Graft-versus-host-disease.

NK cells (choice D) are a type of cytotoxic lymphocyte that constitute a major component of the innate
immune system. NK cells play a major role in the rejection of tumors and cells infected by viruses. They kill
cells by releasing small cytoplasmic granules of proteins called perforin and granzyme that cause the target
cell to die by apoptosis. NK cells are not involved in Graft-versus-host-disease.

Macrophages (choice E) are phagocytes, acting in both non-specific defense (innate immunity) as well as
to help initiate specific defense mechanisms (adaptive immunity). Their role is to phagocytose (engulf and
then digest) cellular debris and pathogens either as stationary or as mobile cells, and to stimulate
lymphocytes and other immune cells to respond to the pathogen. Macrophages are not involved in Graft-
versus-host-disease.

A 55-year-old, obese but previously healthy woman is hospitalized for pneumonia. On the 15th hospital
day she is found to have swelling and tenderness of her right leg which apparently had developed over the
past 48 hours. On her limb examination, straight leg raises elicits pain in the calf muscles. Her leg is red
and warm, but no fever is present. Cardiac, and abdominal examinations are absolutely normal, except
the positive findings for pneumonia on respiratory examination.

An ultrasound examination reveals findings suggestive of femoral vein thrombosis. She has no previous
history of abortions, but she has been using post menopausal hormonal replacement therapy for the last
year.

Which of the following conditions is most likely to have contributed to the appearance of these findings?

Trousseau syndrome
Factor v leiden mutation

Warfarin induced skin necrosis

Disseminated intravascular coagulation,DIC

Prolonged immobilization

Explanation

The correct answer is choice E.

Among many causes of femoral vein thrombosis (deep vein thrombosis, DVT), the most simple cause is
prolonged immobilization. All of the other mentioned causes are related to DVT but not the best answer in
this case.

DVT's are a medical emergency, as there is a 3% chance of a pulmonay embolism that can be fatal in

5
some cases. Causes and risk factors for DVT's include the following:

 An inherited condition, factor V Leiden deficiency causes increased risks for clotting (factor V
Leiden deficiency is most common type of genetic thrombophilia)
 Cancer and its treatment (Trousseau syndrome)
 Low blood flow in a deep vein due to injury, surgery, immobilization due to hospitalization or bed
ridden patients
 Sitting for a long time, such as long trips in a car or on an airplane
 Pregnancy, especially the first six weeks after giving birth (postpartum)
 Taking birth control pills or hormone therapy, including postmenopausal HRT
 Other drugs such as estrogen or erythropoietin
 Other medical conditions, such as varicose veins
 Over the age of 60 (although deep vein thrombosis can occur in any age group)
 Obesity, diabetes and smoking

Prolonged immobilization (choice E) can cause circulatory stasis, which is one of the predisposing factors
for DVT described by Virchow in his famous triad:

 endothelial injury
 stasis of circulation
 hypercoagulability

For travelers, prolonged immobilization is defined as a flight longer than 6-10 hours (most studies use an
average of 8 hours). For nursing home residents, the definition of prolonged immobilization varies between
less than 15 days (increased risk of DVT) and/or longer than 3 months.

In certain cancers, especially adenocarcinomas of the pancreas and lung, associated venous thrombosis
and hypercoagulability is known as trousseau sign (choice A). When thrombophlebitis occurs repeatedly in
different locations, it is known as "migrating thrombophlebitis".

Factor V Leiden deficiency (choice B) is a type of thrombophilia that occurs due to activated protein C
resistance. APC's main role is to degrade Factor Va. APC resistance is the inability of protein C to degrade
Factor Va. This may be hereditary or acquired in some cases. Suspicion of factor V Leiden deficiency should
be considered in any patient below the age of 45, or in any person with a family history of venous
thrombosis. Factor V Leiden deficiency doubles the life-time risk of developing a DVT in an individual.

Warfarin induced skin necrosis (choice C), is a type of thrombophilia which occurs due to protein C
deficiency. This may be inherited or acquired. The skin lesions of WISN occur on the extremities, torso,
breasts, and penis. They begin as erythematous macules and, if appropriate therapy is not initiated
promptly, they may evolve to become purpuric and necrotic bullae.

Disseminated intravascular coagulation, DIC (choice D) is another type of thrombophilia. DIC may be
congenital or acquired. It is characterized by a systemic activation of the blood coagulation system, which
results in the generation and deposition of fibrin, leading to microvascular thrombi in various organs and
contributing to the development of multiorgan failure. Later on in the natural course of DIC, a bleeding
disorder may also develop.

A newborn infant develops respiratory distress immediately after birth. His abdomen is scaphoid. No
breath sounds are heard on the left side of his chest, but they are audible on the right. Immediate
intubation is successful with little or no improvement in clinical status. Emergency chest x-ray is shown
below.

The most likely explanation for this infant’s conditions is which of the following:

6
pneumonia
cyanotic heart disease
diaphragmatic hernia
choanal atresia
pneumothorax

Click on image to Zoom

Explanation

The correct answer is choice C.

Congenital diaphragmatic hernia occurs due presence of developmental defects in the diaphragm through
the bowel loops herniates to chest compressing the lungs and producing pulmonary hypoplasia, pulmonary
hypertension, pulmonary immaturity, and potential deficiencies in the surfactant and antioxidant enzyme
system. In 85% of cases the herniation occurs through a defect on the left side of diaphragm called
foramen of Bochdalek. Lung compression explains the signs seen in this case as respiratory distress and
absent breath sounds on one side. The x-ray show mediastinal shift to the right and air filled bowel in the
left side.

Answer A is false. In pneumonia, auscultation would reveal focal crackles and wheezes and not absence of
breath sounds. X-ray would show the pneumonic patches with no shift of mediastinum.

Answer B is false as cyanotic heart disease would not produce absent of breath sounds but murmurs of the
cardiac lesions will be heard. Also the x-ray would show the anomalies of the cardiac shadow.

Answer D is false. Choanal atresia, a complete nasal obstruction in a newborn, can produce severe
respiratory distress but the signs of diminished air entry into the chest would not be unilateral as in this
condition.

Answer E is false. Pneumothorax can produce a picture similar to this condition but the x-ray in
pneumothorax would show jet-black translucency with collapse of the underlying ling.

A 60-year-old man with a past medical history of primary hyperparathyroidism presents to the emergency
department with nausea, vomiting, constipation, confusion, polyuria, and generalized weakness. The
patient is immediately placed on a monitor, intravenous (IV) access is obtained, and a Foley catheter is
inserted. Initial laboratory testing reveals a serum calcium level of 16 mg/dL.

What is the next step in this patient’s management?

7
IV fluid resuscitation
Bisphosphonates
Calcitonin
Urgent neck and chest CT scan
IV loop diuretic (furosemide)

Explanation

The correct answer is choice A

Hypercalcaemia is an elevated calcium level in the blood ((Normal range: 9-10.5 mg/dL). It can be an
asymptomatic laboratory finding. However, an elevated calcium level is often indicative of other diseases
and a differential diagnosis should be undertaken if it persists. It can be due to excessive skeletal calcium
release, increased intestinal calcium absorption, or decreased renal calcium excretion. Primary
hyperparathyroidism and malignancy account for about 90% of cases of hypercalcemia.

EKG findings of a shortened QT interval and a widened T wave suggest hypercalcaemia and can lead to
abnormal heart rhythms. Finally, peptic ulcers may also occur. Symptoms are more common at high
calcium blood values (12.0 mg/dLl). Severe hypercalcaemia (above 15–16 mg/dL) is considered a medical
emergency: at these levels, coma and cardiac arrest can result. Initial therapy consists of IV fluids and
diuretics (furosemide) supplemented with bisphosphonates and calcitonin.

The general mnemonic for remembering the physical symptoms of hypercalcaemia: "groans (constipation),
moans (psychotic noise), bones (bone pain, especially if PTH is elevated), stones (kidney stones), and
psychiatric overtones (including depression and confusion)." Other symptoms can include fatigue,
anorexia, nausea, vomiting, pancreatitis and increased urination.

The goal of therapy is to treat the hypercalcemia first and subsequently effort is directed to treat the
underlying cause. Initial therapy involves fluids and forced diuresis. Hydration is needed because many
patients are dehydrated due to vomiting or renal defects in concentrating urine. Increased salt intake also
can increase body fluid volume as well as increasing urine sodium excretion, which further increases
urinary calcium excretion. After rehydration, a loop diuretic such as furosemide can be given to permit
continued large volume intravenous salt and water replacement while minimizing the risk of blood volume
overload and pulmonary edema. In addition, loop diuretics tend to depress renal calcium reabsorption
thereby helping to lower blood calcium levels.

Bisphosphonates (choice B) can be used as additional therapy after hydration. Bisphosphonates are taken
up by osteoclasts and inhibit osteoclastic bone resorption. Calcitonin blocks bone resorption and also
increases urinary calcium excretion by inhibiting renal calcium reabsorption

Calcitonin (choice C) can be used as additional therapy after hydration. Calcitonin blocks bone resorption
and also increases urinary calcium excretion by inhibiting renal calcium reabsorption

Neck exploration and removal of the enlarged gland (choice D) is not the first line treatment of an acutely
symptomatic hypercalcemic patient. It is the next best step for definitive treatment after the patient has
stabilized.

A four day old infant delivered at home presents with bilious vomitting and abdominal distention. He has not passed
meconium since birth. You suspect a case of Hirschsprung's disease.

All of the following statements are true except:

8
Often presents with neonatal large bowel obstruction
Results from absence of ganglion cells in both the
Meissner's and Auberbach's plexus
A contrast-study will show dilatation of the aganglionic
segment
Early treatment may involve rectal irrigation or an
emergency colostomy
Associated with trisomy 21

Click on image to Zoom

Explanation

The correct answer is Choice C

Hirschsprung’s disease is also known as aganglionic megacolon, a congenital disease caused by a failure of
parasympathetic ganglion cells to migrate into the hindgut resulting in a functional obstruction of the large
intestine. Both the myenteric (Auerbach) and the submucosal (Meissner) plexus are absent (Choice B)
resulting in reduced bowel peristalsis. Absence of parasympathetic innervation begins in the anus and
extends proximally for a variable distance causing functional obstruction at the junction (transition zone)
between normal bowel and the distal aganglionic bowel. In 80% of cases, the lesion is confined to the
recto-sigmoid colon (short-segment disease) and in 20% of cases, the entire colon is involved (long-
segment disease).

The incidence of Hirschsprung’s is 1 in 5000 live births, occurring three times more commonly in males
than females. The condition has been associated with Down’s syndrome (trisomy 21) (Choice E) and
multiple endocrine neoplasia type II. There is a positive family history in 10% of cases. 99% of normal
infants pass meconium within 24 hours of birth, and Hirschsprungs disease usually presents within the first
few days of life with features of large bowel obstruction (Choice A) including failure to pass meconium,
abdominal distension and bile-stained vomiting. The diagnosis is suspected clinically, but can usually be
confirmed by;

 Plain abdominal x-rays; may show distended loops of bowel with reduced air in the rectum
 Barium enema; contrast studies will show a constricted, NOT dilated distal aganglionic segment (Choice C), a
narrow transition zone and a dilated proximal bowel segment
 Rectal biopsy; obtained under anaesthesia using punch or suction biopsy. Positive biopsy will show the absence
of ganglion cells confirmed by staining for anticholinesterase
 Anorectal manometry; demonstrates absence on normal inhibitory reflex of the internal sphincter on rectal lumen
distension

Differential diagnosis includes anorectal malformations, intestinal atresia or stenosis and intestinal motility
disorders.

Conservative medical treatment is used to treat complications and institute temporary measures until
definitive reconstructive surgery is available. Colonic lavage using mechanical irrigation may be required.
Initial surgical treatment includes creating a diverting colostomy at the time of diagnosis (Choice D).

Definitive surgery is usually performed when the child grows and weighs more than 10 kg.

You are a senior resident in pediatric surgery and you get a call from the neonatologist about a neonate
having severe respiratory distress and cyanosis. On examination you found a scaphoid abdomen, barrel-
shaped chest and signs of respiratory distress including retractions and grunting. Auscultation of chest
revealed diminished air entry on the left side. An orogastric tube was introduced then x-ray film was

9
made. The tube was seen coiled in the left side of chest. Your provisional diagnosis was Congenital
diaphragmatic hernia.

Regarding Congenital diaphragmatic hernias, all of the following statements are true except:

Usually occur through the foramen of Bochdalek


Are more common on the left than the right
Are frequently associated with other congenital
anomalies
First diagnosis can be only made at birth
Neonates usually require sedation, ventilation and
intestinal decompression prior to surgery

Click on image to Zoom

Explanation

The correct answer is choice D.

Congenital diaphragmatic hernia can be diagnosed during prenatal period using ultrasonography with high
sensitivity. The peristalsis of bowel loops can be seen in chest. In addition there may be other suggesting
criteria as polyhydramnios, intrathoracic gastric air bubble and a mediastinal shift away from the
herniation.

Choice A is a true phrase as in 85% of cases herniation occurs through foramen of Bochdalek, a congenital
defect in diaphragm in its left postero-lateral aspect due to incomplete closure of pericardio peritoneal
canal. The remaining 15% includes hernia through foramen of Morgagni, congenital hiatus hernia and
eventration of diaphragm.

Choice B is a true phrase as foramen of Bochdalek is present on the left side of diaphragm and herniation
through it represents 85% of cases as mentioned.

Choice C is a true phrase as there are many congenital anomalies that may be associated with this type of
hernia. 30% of cases are associated with chromosomal anomalies as triosomy 18 or 21 or monosomy x
(Turner syndrome). 10% of cases are syndromatic as Fryns syndrome or Cornelia de Lang syndrome. 25%
of cases are associated with cardiac anomalies as atrial septal defect or transposition of great vessels. Also
there are many neural, urological or pulmonary defects that may be present.

Choice E is a true phrase as once the case is diagnosed, an orogastric tube should be introduced to deflate
stomach and bowel loops allowing lung expansion followed by mechanical ventilation. Sedation is also
required to decrease swallowing of air and allow adequate mechanical ventilation.

A newborn infant presented with repeated bilious vomiting, delayed passage of meconium, scaphoid
abdomen with epigastric fullness. An erect plain x-ray film was done revealing double-bubble sign. The
case was diagnosed as duodenal atresia.

Duodenal atresia is most likely to occur in which of the following syndrome:

10
Down syndrome
Cushing's syndrome
Turner's syndrome
Klinefelter's syndrome
all of the above

Click on image to Zoom

Explanation

The correct answer is A.

Approximately 30% of cases of duodenal atresia are associated with Down syndrome.

Down syndrome (Triosomy 21) is a chromosomal disorder caused by presence of extra 21st chromosome.
The typical phenotype of the Down syndrome includes mental retardation, characteristic facial features,
hand anomalies and congenital heart defects.

Answer B is false. Cushing syndrome is caused by prolonged exposure to high levels of glucocorticoids
whether endogenous or exogenous. Its main criteria include muscle weakness, weight gain, hirsutism,
hypertension, diabetes mellitus and diminished immunity.

Answer C is false. Turner syndrome is a chromosomal disorder caused by absence of one x-chromosome in
a female. The karyotype is 45,x0. The typical phenotype includes short stature, absence of breast
development of breasts, high-arched palate and webbed neck.

Answer D is false. Klinefelter syndrome is a chromosomal disorder caused by presence of extra sex
chromosome. The karyotype is 47,xxy. The typical phenotype includes hypogonadism and gynecomastia in
males.

You were called to examine a newborn infant that presented with bilious vomiting, abdominal distension
and failure to pass meconium. Double bubble sign was seen on a plain erect X-ray.

This picture may suggest the diagnosis of which of the following:

congenital megacolon
Meckel's diverticulum
necrotizing enterocolitis
hypertrophic pyloric stenosis
duodenal atresia

11
Click on image to Zoom

Explanation

The correct answer is choice E.

Duodenal atresia is a congenital anomaly caused by failure of recanalization of the epithelium of the
duodenum. Also annular pancreas produce a similar condition as the pancreatic tissue surrounds the entire
circumference of the duodenum compressing it. In both cases, it is presented by non-bilious vomiting as
the obstruction occurs proximal to the ampulla of Vater, failure of passage of meconium and abdominal
distention. Double bubble sign appears in plain erect x-ray films. The first bubble corresponds to the
stomach and the second to the postpyloric and prestenotic dilated duodenal loop.

Answer A is false. Congenital megacolon or Hirschsprung disease is caused by congenital absence of


ganglion cells in the distal alimentary tract. It is usually presented by bilious vomiting and the x- ray show
multiple loops of dilated small bowel with air-fluid levels that can usually be determined to be a distal
bowel obstruction.

Answer B is false as Meckel diverticulum is presented mainly by lower GI bleeding. Intestinal obstruction
occurs in complicated cases only and in such case the vomiting is bilious and the double bubble sign is
absent.

Answer C is false. Necrotizing enterocolitis is not presented by double bubble sign. The main presentation
includes abdominal distension, frank or occult blood in the stools vomiting, diarrhea, feeding intolerance
and high gastric residuals following feedings.

Answer D is false as Hypertrophic pyloric stenosis is presented by a single bubble sign as the obstruction in
this case occurs at the pylorus, proximal to the duodenum.

A 75-year-old man from China presents to GI clinic complaining of increased abdominal girth, weight loss,
and fever for the past year. He has been increasing nauseous for the past month to the point that he can
not hold down food anymore without vomitting. Physical exam is remarkable for right upper quadrant
tenderness and marked hepatomegaly. He is visibly jaundiced. Abdominal ultrasound reveals gall stones
and 3.7 cm common bile duct. His lab results are as follows:

 AST 59 (N<30)
 ALT 37 (N<35);
 ALP 320 (N<120)
 Conjugated bilrubin 70 (N<5);
 Unconjugated bilrubin 10 (N<15)

Which one of the following is most likely to account for his symptoms?

sclerosing cholangitis
common bile duct stone
adenocarcinoma of the head of the pancreas
acute cholestasis
hepatocellular carcinoma

Explanation

12
The correct answer is choice E

This patient's symptoms are consistent with a hepatic process. Hepatocellular carcinoma (HCC) is a
primary malignancy of the liver. Most cases of HCC are secondary to either a viral hepatitide infection
(hepatitis B or C) or cirrhosis (alcoholism being the most common cause of hepatic cirrhosis). Of note,
China is an area of endemic Hepatitis B infection.

HCC may present with jaundice, bloating from ascites, easy bruising from blood clotting abnormalities, loss
of appetite, unintentional weight loss, abdominal pain,especially in the upper -right part, nausea, emesis,
or fatigue.

Hepatocellular carcinoma (HCC) most commonly appears in a patient with chronic viral hepatitis (hepatitis
B or hepatitis C, 20%) or with cirrhosis (about 80%). These patients commonly undergo surveillance with
ultrasound due to the cost-effectiveness. In patients with a higher suspicion of HCC (such as rising alpha-
fetoprotein), the best method of diagnosis involves a CT scan of the abdomen using intravenous contrast
agent and three-phase scanning (before contrast administration, immediately after contrast administration,
and again after a delay) to increase the ability of the radiologist to detect small or subtle tumors. It is
important to optimize the parameters of the CT examination, because the underlying liver disease that
most HCC patients have can make the findings more difficult to appreciate.

Surgical resection offers the best prognosis for long-term survival but only 10-15% of patients are suitable
for surgical resection. This is often due to extensive disease or poor liver function. Resection in cirrhotic
patients carries high morbidity and mortality. The overall recurrent rate after resection is 50-60%.

Sclerosing cholangitis (choice A) is a chronic liver disease caused by progressive inflammation and scarring
of the bile ducts of the liver. The inflammation impedes the flow of bile to the gut, which can ultimately
lead to liver cirrhosis, liver failure and liver cancer. The underlying cause of the inflammation is believed to
be autoimmunity. The diagnosis is by imaging of the bile duct, usually in the setting of endoscopic
retrograde cholangiopancreatography (ERCP, endoscopy of the bile duct and pancreas), which shows
"beading" (both strictures and dilation) of the intrahepatic and extrahepatic bile ducts.

Choledocholithiasis (choice B) is the presence of gallstones in the common bile duct. This condition causes
jaundice and liver cell damage, and is a medical emergency, requiring the endoscopic retrograde
cholangiopancreatography (ERCP) procedure or surgical treatment. This obstruction may lead to jaundice,
elevation in alkaline phosphatase, increase in conjugated bilirubin in the blood and increase in cholesterol
in the blood. It can also cause acute pancreatitis and ascending cholangitis. The common bile duct is
normal in size so a stone is unlikely.

Adenocarcinoma of the head of the pancreas (choice C) is sometimes called a "silent killer" because early
pancreatic cancer often does not cause symptoms, and the later symptoms are usually nonspecific and
varied. Therefore, pancreatic cancer is often not diagnosed until it is advanced. Common symptoms include
pain in the upper abdomen that typically radiates to the back (seen in carcinoma of the body or tail of the
pancreas), loss of appetite and/or nausea and vomiting, significant weight loss, painless jaundice (yellow
skin/eyes, dark urine) when a cancer of the head of the pancreas (about 60% of cases) obstructs the
common bile duct as it runs through the pancreas. This may also cause pale-colored stool and steatorrhea.

 Trousseau sign, in which blood clots form spontaneously in the portal blood vessels, the deep
veins of the extremities, or the superficial veins anywhere on the body, is sometimes associated
with pancreatic cancer.
 Diabetes mellitus, or elevated blood sugar levels. Many patients with pancreatic cancer develop
diabetes months to even years before they are diagnosed with pancreatic cancer, suggesting new
onset diabetes in an elderly individual may be an early warning sign of pancreatic cancer.

Liver function tests can show a combination of results indicative of bile duct obstruction (raised conjugated
bilirubin, γ-glutamyl transpeptidase and alkaline phosphatase levels). CA19-9 (carbohydrate antigen 19.9)
is a tumor marker that is frequently elevated in pancreatic cancer. Elevated liver enzymes, however, are
not often elevated in pancreatic cancer confined to the head of the pancreas.

Cholestasis (choice D) is a condition where bile cannot flow from the liver to the duodenum. The two basic

13
distinctions are an obstructive type of cholestasis where there is a mechanical blockage in the duct system
such as can occur from a gallstone or malignancy and metabolic types of cholestasis which are
disturbances in bile formation that can occur because of genetic defects or acquired as a side effect of
many medications.

Acute cholestasis can be suspected when there is an elevation of both 5'-nucleotidase and ALP enzymes.
With a few exceptions, the optimal test for cholestasis would be elevations of serum bile acid levels. The
gamma-glutamyl transferase(GGT) enzyme was previously thought to be helpful in confirming a hepatic
source of ALP; however, GGT elevations are markedly sensitive and lack the necessary specificity to be a
useful confirmatory test for ALP. Normally GGT and ALP are anchored to membranes of hepatocytes and
are released in small amounts in hepatocellular damage. In cholestasis, synthesis of these enzymes is
induced and they are made soluble. GGT is elevated because it leaks out from the bile duct cells due to
pressure from inside bile duct.

In a later stage of cholestasis AST, ALT and bilirubin may be elevated due to liver damage as a secondary
effect of cholestasis but not in the acute phase.

On postoperative day two of an uncomplicated sigmoid colon resection for cancer, a 71 yr old man is
noted to be short of breath and tachycardic. His vitals are within normal limits with the exception of
temperature of 37.6 °C. Fine crackles are heard at the bases of both lungs. His medical history is
significant for hypertension, hyperlipidemia, and carotid stenosis. He has smoked a pack of cigarretes for
the past thirty years. A portable chest radiograph is taken.

Which one of the following is most likely to account for the patient’s postoperative symptoms?

atelectasis
pneumonia
pulmonary embolism
myocardial infarction
pneumothorax

Click on image to Zoom

Explanation

The correct answer is choice A

Atelectasis is defined as the lack of gas exchange within alveoli, due to alveolar collapse or fluid
consolidation. It may affect part or all of one lung. It is a condition where the alveoli are deflated, as
distinct from pulmonary consolidation.

It is a very common finding in chest x-rays and other radiological studies. It may be caused by normal
exhalation or by several medical conditions. Although frequently described as a collapse of lung tissue,
atelectasis is not synonymous with a pneumothorax, which is a more specific condition that features
atelectasis. Acute atelectasis may occur as a post-operative complication or as a result of surfactant
deficiency. In premature neonates, this leads to infant respiratory distress syndrome.

Atelectasis may be an acute or chronic condition. In acute atelectasis, the lung has recently collapsed and
is primarily notable only for airlessness. In chronic atelectasis, the affected area is often characterized by a

14
complex mixture of airlessness, infection, widening of the bronchi (bronchiectasis), destruction, and
scarring (fibrosis)

Acute atelectasis is a common postoperative complication, especially after chest or abdominal surgery. The
most common cause is post-surgical atelectasis, characterized by splinting, restricted breathing after
abdominal surgery. Smokers and the elderly are at an increased risk. Acute atelectasis may also occur with
an injury, usually to the chest (such as that caused by a car accident, a fall, or a stabbing). Atelectasis
following surgery or injury involves most alveoli in one or more regions of the lungs. In these
circumstances, the degree of collapse among alveoli tends to be quite consistent and complete. Large
doses of opioids or sedatives, tight bandages, chest or abdominal pain, abdominal swelling (distention),
and immobility of the body increase the risk of acute atelectasis following surgery or injury, or even
spontaneously.

In acute atelectasis that occurs because of a deficiency in the amount or effectiveness of surfactant, many
but not all alveoli collapse, and the degree of collapse is not uniform. Atelectasis in these circumstances
may be limited to only a portion of one lung, or it may be present throughout both lungs. When premature
babies are born with surfactant deficiency, they always develop acute atelectasis that progresses to
neonatal respiratory distress syndrome. Adults can also develop acute atelectasis from excessive oxygen
therapy and from mechanical ventilation.

pneumonia (choice B) is an inflammatory condition of the lung. It is often characterized as including


inflammation of the parenchyma of the lung and abnormal alveolar filling with fluid (consolidation and
exudation). Although pneumonia is a postoperative complication and a cause of postoperative fever,
patients will present with infiltrate on chest x-ray with fever > 37.8 °C (100.0 °F), tachycardia,
rales/crackles, and decreased breath sounds.

pulmonary embolism (choice C) is a blockage of the main artery of the lung or one of its branches by a
substance that has travelled from elsewhere in the body through the bloodstream (embolism). Usually this
is due to embolism of a thrombus (blood clot) from the deep veins in the legs, a process termed venous
thromboembolism. A small proportion is due to the embolization of air, fat or amniotic fluid. The
obstruction of the blood flow through the lungs and the resultant pressure on the right ventricle of the
heart leads to the symptoms and signs of PE. On chest radiograph, PE manifests as a a wedge shaped,
pleural based consolidation associated with pulmonary infarction (Hampton's hump) or a focus of oligemia
(vasoconstriction) seen distal to a pulmonary embolus (Westermark Sign).

myocardial infarction (choice D) , commonly known as a heart attack, is the interruption of blood supply to
part of the heart, causing heart cells to die. MI can be a common cause of postoperative chest pain and
shortness of breath. However, patients with an acute myocardial infarction may have a completely normal
chest radiograph.

pneumothorax (choice E) is a collection of air or gas in the pleural cavity of the chest between the lung and
the chest wall. Medical procedures of the chest (iatrogenic), such as the taking of biopsy samples from
lung tissue, inserting a central venous catheter into one of the chest veins, may lead to injury to the lung
and resultant pneumothorax. Traditionally a plain radiography of the chest, ideally with the X-ray beams
being projected from the back (posteroanterior or PA), has been the most appropriate first investigation.
The size of the pneumothorax, i.e. the amount of space in the chest taken up by free air rather than air-
containing lung, can be determined with a reasonable degree of accuracy by measuring the distance
between the chest wall and the lung.

A 2 year old infant presented with asymptomatic nodule over the suprasternal notch. It was biopsied and
eventually removed for histological confirmation.

Which of the following is the most accurate diagnosis?

15
Branchial cleft cyst
Bronchogenic cyst
Midline cervical cleft
Cervical thymic cyst
Thyroglossal cyst

Click on image to Zoom

Explanation

The correct answer is B.

A bronchogenic cyst is a congenital anomaly that develops due to abnormal budding of the foregut during
fetal life. It is the second most common subtype of foregut cysts found in the middle mediastinum. More
than 50% of cases are diagnosed in patients older than 15 years.

In 85% of cases the cyst is located within the mediastinum. In this case, it presented with an
asymptomatic cystic mass at the suprasternal notch. It may be associated with fever if inflamed. The most
dangerous complication of such cysts is the mass effect:

 Compression of the GI tract can result in dysphagia.


 Compression of the airway, especially if cyst is just below the carina, can result in life-threatening
respiratory distress.
 Compression of the heart and great vessels can result in dysrhythmias and obstruction of the
vena cava.

Answer A is false. A branchial cleft cyst is a congenital anomaly caused by failure of obliteration of the 2nd
branchial cleft. It is presented by a unilateral cyst usually in front of the anterior border of the
sternomastoid muscle.

Answer C is false. Midline cervical cleft is a very rare congenital anomaly that is thought to be due to
abnormal fusion between the 2nd and 3rd branchial arches. It is presented as a cleft extending from the
suprasternal notch.

Answer D is false. Thymic cyst is a sequestration cyst arising from the thymopharyngeal duct at any point
of its course from the level of the mandible to the chest. However it is an extremely rare condition
especially when compared to the frequency of the bronchogenic cysts.

Answer E is false. Thyroglossal cyst occurs due to failure of obliteration of a part of the thyroglossal duct.
In most cases it is present just below the hyoid bone and over the thyroid cartilage. In very few cases it
may be present at the suprasternal notch.

A 63-year-old alcoholic man presents to the Emergency Department with increasing abdominal girth and
right upper quadrant pain. On physical exam, the patient has a narrow pulse pressure and an S3 and S4
are heard on ascultation of the heart. Examination of the right neck veins reveals venous pulsations that
are best seen when the head of the bed is elevated to ninety degrees. The abdomen is distended with
shifting dullness and riddled with spider veins.

The synthetic function of the liver is measured by which of the following parameters according to the
Child’s classification?

16
ascites levels
elevated serum transaminases (ALT, AST)
bilirubin diglucuronide levels
serum albumin and prothrombin time
Hepatic encephalopathy (ammonia levels)

Explanation

The correct answer is choice D

Cirrhosis is a consequence of chronic liver disease characterized by replacement of liver tissue by fibrosis,
scar tissue and regenerative nodules,leading to loss of liver function. Cirrhosis is most commonly caused
by alcoholism, hepatitis B and C, and fatty liver disease but has many other possible causes. Some cases
are idiopathic. Cirrhosis is often preceded by hepatitis and fatty liver (steatosis), independent of the cause.
If the cause is removed at this stage, the changes are still fully reversible.

The pathological hallmark of cirrhosis is the development of scar tissue that replaces normal parenchyma,
blocking the portal flow of blood through the organ and disturbing normal function. Recent research shows
the pivotal role of the stellate cell, a cell type that normally stores vitamin A, in the development of
cirrhosis. Damage to the hepatic parenchyma leads to activation of the stellate cell, which becomes
contractile (called myofibroblast) and obstructs blood flow in the circulation. In addition, it secretes TGF-
β1, which leads to a fibrotic response and proliferation of connective tissue. Furthermore, it disturbs the
balance between matrix metalloproteinases and the naturally occurring inhibitors, leading to matrix
breakdown and replacement by connective tissue-secreted matrix. The fibrous tissue bands (septa)
separate hepatocyte nodules, which eventually replace the entire liver architecture, leading to decreased
blood flow throughout. The spleen becomes congested, which leads to hypersplenism and increased
sequestration of platelets. Portal hypertension is responsible for most severe complications of cirrhosis.

The Child-Pugh score is used to assess the prognosis of chronic liver disease, mainly cirrhosis. Although it
was originally used to predict mortality during surgery, it is now used to determine the prognosis, as well
as the required strength of treatment and the necessity of liver transplantation. Hepatic decompensation
and failure may be predicted by identifying patients with jaundice and marginal liver hepatic synthetic
function.

The score employs five clinical measures of liver disease:

 Bilirubin (total) <34


 Serum albumin >35
 INR <1.7
 Ascites
 Hepatic encephalopathy

17
The liver plays a vital role in synthesis of proteins (e.g., albumin, clotting factors and complement),
detoxification and storage (e.g., vitamin A). In addition, it participates in the metabolism of lipids and
carbohydrates. Therefore, a decrease in albumin and clotting factors signify a a decreased synthetic liver
function in the Child’s scheme.

Ascites (choice A) is an accumulation of fluid in the peritoneal cavity. Ascitic fluid can accumulate as a
transudate or an exudate. Amounts of up to 25 liters are possible. Roughly, transudates are a result of
increased pressure in the portal vein (>8 mmHg, usually around 20 mmHg), e.g. due to cirrhosis, while
exudates are actively secreted fluid due to inflammation or malignancy. The presence of ascites provides
no information on synthetic liver function status.

Elevated serum transaminases (choice B) , commonly the transaminases alanine transaminase (ALT) and
aspartate transaminase (AST), may be an indicator of liver damage. The liver has a variety of
transaminases to synthesize and break down amino acids and to interconvert energy storage molecules.
The concentrations of these in the serum are normally low. However, if the liver is damaged, the
hepatocyte cell membrane becomes more permeable and some of the enzymes leak out into the blood
stream. Elevated serum transaminases provide no information on synthetic liver function status.

Bilirubin diglucuronide (choice C) is a conjugated form of bilirubin. In the liver it is conjugated with
glucuronic acid by the enzyme Glucuronyltransferase, making it soluble in water. Much of it goes into the
bile and thus out into the small intestine. Some of the conjugated bilirubin remains in the large intestine
and is metabolised by colonic bacteria to urobilinogen, which is further metabolized to stercobilinogen, and
finally oxidised to stercobilin. This stercobilin gives feces its brown color. Some of the urobilinogen is
reabsorbed and excreted in the urine along with an oxidized form, urobili. Bilirubin diglucuronide provides
no information on synthetic liver function status.

Hepatic encephalopathy (choice E) is the occurrence of confusion, altered level of consciousness and coma
as a result of liver failure. It is caused by accumulation of ammonia in the bloodstream that are normally
removed by the liver. The diagnosis of hepatic encephalopathy requires the presence of impaired liver
function and the exclusion of an alternative explanation for the symptoms. Blood tests (ammonia levels)
may assist in the diagnosis. Attacks are often precipitated by an intercurrent problem, such as infection or
constipation. Hepatic encephalopathy provides no information on synthetic liver function status.

You are seeing a 52 year old male patient in the office who smokes three packs of cigarettes per day and
has smoked for over 35 years. He has lost 15 lbs during the past four months without being on a diet. He
also complains about on going shortness of breath for the past 2 months. His symptoms seem to worsen
despite the bronchodilators that you have prescribed. He has a history of chronic cough but rarely
productive.

A few days later, he presented to the emergency due to worsening shortness of breath. On physical

18
examination, he was tachypneic with bilateral upper extremity swelling along with facial puffiness. There
was no evidence of cyanosis and his oxygen saturation was 92% on 2L oxygen. His respiratory exam was
normal and cardiac exam showed normal heart sounds a and JVP was approximately 2 cm above the
sternal angel. His abdomen was soft and nontender.

The most likely cause of these findings is which of the following:

congestive heart failure


chronic obstructive pulmonary disease (COPD) and emphysema
obstruction of the superior vena cava
pulmonary embolus
thrombosis of the subclavian vein

Explanation

The correct answer is choice C.

More than 90% of patients who present with superior vena cava syndrome, as in this patient, have an
associated malignancy (causing weight loss).

Superior vena cava (SVC) syndrome (choice C) is characterized by gradual compression/obstruction of the
superior vena cava, with a gradual increase in symptomatology. The thin walled superior vena cava is
surrounded by rigid structures and is relatively easy to compress. Low intravascular pressure also allows
for the possibility of thrombus formation.

SVCS is chiefly associated with malignancies, as the Infectious causes (e.g. syphilis, tuberculosis) have
decreased because of improvements in antibiotic therapy. Commonly associated malignancies include
bronchogenic carcinoma among other lung cancers, lymphomas, and metastatic malignant diseases.
Clinical presentations with a history of thoracic malignancy often support the diagnosis of SVCS. Plain
radiography is helpful and may reveal a mediastinal mass in many patients. Symptoms most often include:

 Facial swelling and cyanosis


 Cough
 Chest discomfort
 Dyspnea
 Fever
 Weight loss
 Blurred vision

Signs most often include:

 Swelling of upper extremities and face


 Superficial vein dilatation
 Lymphadenopathy
 Hoarseness
 Horner’s syndrome

Chronic obstructive pulmonary disease or COPD (option B), is a mixture of 3 separate disease processes.
The patient has symptoms of this classic triad, which includes:

19
 chronic bronchitis
 emphysema
 asthma (but to a lesser extent)

Signs and symptoms of chronic bronchitis (blue bloaters):

 Patients may be obese


 Frequent productive cough
 Intermittent dyspnea
 Recurrent pulmonary infections
 Cardiac/respiratory failure over time, with edema and weight gain
 Coarse rhonchi and wheezing may be heard on auscultation

Signs and symptoms of emphysema (pink puffers):

 Patients may be very thin with a barrel chest.


 Little or no cough or expectoration
 Progressive dyspnea
 The chest may be hyperresonant, and wheezing may be heard; heart sounds are very distant.
 Cachexia (wasting syndrome causing loss of weight, muscle atrophy, fatigue, weakness)
 Respiratory failure

Congestive heart failure, CHF (choice A) may present as left sided, right sided or biventricular failure.
Patient with left sided failure usually have dyspnea on exertion or even at rest. In right sided failure
peripheral, pitting edema occurs. This does not match the scenario.

Pulmonary embolism (PE) (choice D) does not fit this clinical scenario.

The primary etiology of thrombosis of subclavian vein (choice E) is referred to as effort-induced thrombosis
or Paget-von Schrötter syndrome. It usually results from the excessive use of the involved arm. External
compression of the axillary-subclavian vein has been suggested to contribute to the stasis of blood that
leads to thrombosis. The most common lung malignancy associated with subclavian thrombosis has been
the Pancoast tumor which presents with the following:

 Mild-to-moderate non-pitting edema


 Mild cyanosis of the hands and fingers on the affected side (whereas, bilateral swelling in cases of
SVCS)
 Dilatation of subcutaneous collateral veins may be present over the upper arm and chest
 In a few cases, in which the diagnosis was missed or delayed or the patient presented late, the
thrombus may have extended to the superior vena cava. These patients show most features of
superior vena cava syndrome, including face and neck swelling, periorbital edema, blurred vision,
and some degree of facial cyanosis.

Meckel's diverticulum is the most common congenital anomaly of small intestine. It is present in about 2%
of population located 50-60 cm proximal to the ileo-cecal valve. It is cased by incomplete obliteration of
the vitelline duct.

Regarding Meckel’s diverticulum, all of the following statements are correct except:

20
When hemorrhage complicates a Meckel’s
diverticulum, blood transfusion is often necessary
Bleeding from a Meckel’s diverticulum is the
commonest cause of GI hemorrhage in the pediatric
population
Meckel’s diverticulum is the commonest cause in
pediatric cases of intussusception.
Meckel’s diverticula do not contain ectopic tissue.
All of the above are false
All of the above are true

Click on image to Zoom

Explanation

The correct answer is choice E.

None of these phrases is true.

Answer A is a false phrase. Meckel’ diverticulum often contains ectopic gastric tissue the secretes gastric
acid leading to ulceration of the surrounding tissues. The bleeding from these ulcers is the commonest
presentation of this case but it is usually not severe and does not require blood transfusion.

Answer B is false as the commonest causes of GI bleeding in pediatrics include oesophagitis, gastritis,
duodenitis, intussusception, colonic polyp, and anorectal disorders rather than Meckel’s diverticulum.

Answer C is false as the commonest leading point of intussusception is enlarged Peyer patches due to
adenovirus infection (95% of cases).

Answer D is false as heterotropic gastric mucosa was found in 62% of cases, pancreatic tissue was found in
6%, both pancreatic tissue and gastric mucosa were found in 5%, jejunal mucosa was found in 2%.
Rarely, colonic, rectal, endometrial, and hepatobiliary tissues have been noted.

A 5-years old child presented with unilateral painless mass in the neck. On examination, the mass was
cystic, non-tender with smooth surface and located in front of and superficial to the lower third of the
anterior border of sternomastoid muscle. Your provisional diagnosis was a branchial cyst.

A branchial cyst develops from the vestigial remnants of which of the following:

21
2nd branchial cleft
3rd branchial cleft
1st branchial cleft
4th branchial cleft
5th branchial cleft

Click on image to Zoom

Explanation

The correct answer is choice A

The branchial apparatus, which is formed during early gestation, consists of six mesodermal branchial
arches separated externally by five ectodermal branchial clefts and internally by five endodermal branchial
pouches. The second arch grows caudally to meet the fifth arch. This causes the second, third and the
fourth clefts to form one cavity called the cervical sinus of His. Normally this sinus obliterates during the
fetal life.

A branchial cyst develops due to failure of obliteration of the second branchial cleft. 95% of the branchial
anomalies are due to abnormalities of this particular cleft.Usually it is presented as asymptomatic neck cyst
but may enlarge and become tender when infected. Patients also may present with a mass effect such as
difficulty in breathing. Complete surgical resection is the treatment of choice and results in a good
prognosis. Complications of surgical treatment include recurrence, formation of a persistent fistula, and
damage to the cranial nerves. Patients with infected cysts receive a full course of antibiotics before surgery
to decrease the risk of recurrence and persistent fistula.

Answer B is false as the third branchial cleft rarely gives rise to a cyst and in this case, the cyst usually
lies deep to the sternomastoid muscle.

Answer C is false as the first cleft develops into the external auditory meatus and it is not related to any
structures in the neck.

Answer D is false as the cysts arising from the fourth cleft are extremely rare and they are usually located
inside the thyroid gland or within the mediastinum.

Answer E is false as the fifth is almost always a rudimentary structure.

Predisposing factors increasing risk of infection after surgical inervention are the following EXCEPT:

Mechanical ventilation use


Previous aminoglycoside therapy
Peritonitis
Laparoscopic technique
Longer hospital stay and APACHE II score on admission

22
Explanation

The correct answer is Choice D.

Nosocomial infections are major consideration in mortality rate in surgical intensive care units. High
incidence of resistance to multiple antibacterials and surgical complications contribute to this rate. In the
United States, about 2-5% of patients undergoing surgical procedure develop surigical site infection (SSI).
Overall SSI rate was found to be significantly lower for laparoscopic technic for cholecystectomy (0.62%)
than for the open alternative of gallbladder surgery (1.82%). SSI risk was decreased in patients
undergoing the laparoscopic technique than with the open technique (Choice D).

Bloodstream infections and ventilator-assisted pneumonia were the most common type of infections and
increase mortality rate. The most common organisms identified in several studies are the following:

 Acinetobacter baumannii
 Pseudomonas aeruginosa
 Candida albicans
 Enterococcus faecalis
 Klebsiella pneumoniae
 Enterococcus faecium
 Staphylococcus aureus

Multi-drug resistance to infection, of which previous aminoglycoside therapy contributes to, can be linked
to more severe medical comorbid conditions and contributes to the high incidence of nosocomial infection
(Choice B).

Due to complications of contamination, peritonitis and abdominal surgery increase the risk of bloodstream
infection and bacterial seeding within the abdominal cavity (Choice C).

The APACHE II or Acute Physiology and Chronic Health Evaluation II is a measure of severity of disease
classification system used as one of the scoring systems in the intesive care setting. The higher the
APACHE II score, the more severe the disease and the higher the mortality rate (Choice E).

A 2 year old child complaining of recurrent attacks of colicky abdominal pain, recurrent non-
bilious vomiting which subsequently became bilious and who also developed melena and mucus discharge
per rectum. On examination, there was a sausage-shaped mass in right hypochondrium. The case was
diagnosed as intussusception.

Regarding intussusception, all of the following statements are true except:

23
It is the commonest cause of intestinal obstruction
between 6 and 18 months
Usually occurs in the jejunum
The lead point may be a Peyer's patch or Meckel's
diverticulum
Has a characteristic 'doughnut' appearance on
ultrasound
Red currant jelly is a characteristic sign

Click on image to Zoom

Explanation

The correct answer is B.

Intussusception is a process in which a loop of intestine invaginates into the lumen of an adjacent
intestinal loop.

In most cases it is idiopathic but it is thought to occur due to lymphoid tissue hypertrophy in response to a
rotavirus infection acting as a leading point for intussusception.

In 90% of cases it occurs at ileo-colic region as it has the highest concentration of lymph nodes.

Answer A is a true phrase as most cases of intussusception is idiopathic and occurring at this specific group
of age.

Answer C is true phrase as lymph tissue of Peyer`s patch and is liable to hypertrophy with infection acting
as a leading point inducing vigorous peristalsis. Also the mouth of Meckel`s diverticulum acts by the same
mechanism.

Answer D is a true phrase. Doughnut (ring-like) appearance seen in longitudinal view of ultrasonography is
made by the multiple layers representing the walls of intussuscepted bowel loops.

Answer E is a true phrase. Red currant jelly is formed by sloughed mucosa (mucosa of intussuscepted loop
become ishaemic and sloughed), blood and mucus.

A 49-year-old woman comes to your office and complains of significant pain in her lower extremities that
develops every time she ambulates more than a few blocks (300 meters). This has been ongoing for the
past 5 months and seems to be worsening with time. Her pain relieved after rest and redevelops
immediately after she continues to ambulate. Over the last 3 days, she has noticed numbness that has
developed on her bilateral dorsal feet. On history, she is a smoker but denies any history of diabetes or
vascular disease. Her body mass index is 32.

On physical examination, her lower extremities are cool and pale, without swelling, tenderness or
erythema. No dorsalis pedis or posterior tibial pulses are palpable. Other systemic findings are normal.

Which of the following abnormalities of the vasculature is most likely to account for these findings?

24
Lymphatic obstruction
Arteriolosclerosis
Atherosclerosis
Medial calcific sclerosis
Venous thrombosis

Explanation

The correct answer option is C.

The main concept of the question is to assess your knowledge about arterial, venous and lymphatic
diseases of lower limb.

Arteriolosclerosis (an arterial disease) (choice B) is the hardening and loss of elasticity of small arteries and
arterioles. Types of arteriosclerosis include :

 Athersclerosis (also known as arteriosclerotic vascular disease or ASVD)


 Arteriosclerosis obliterans
 Medial calcific sclerosis (Monckeberg’s calcific sclerosis)

Atherosclerosis (also known as arteriosclerotic vascular disease or ASVD) (choice C) is a condition in which
an artery wall thickens as the result of a build-up of fatty materials such as cholesterol. This is the leading
cause of occlusive arterial disease of the lower extremities. The site of claudication is distal to the location
of the narrowed (stenotic) segment.

The most common presenting symptom is intermittent claudication as in this particular case. The clinical
term is used to describe muscle pains, aches, cramps, numbness or sense of fatigue. These symptoms
come and go with exertion and rest. With more severe disease, pain or numbness of the foot occurs at
night when in a non-dependent position. Symptoms improve when the foot is placed in a dependent
position. In severe claudication, the pain is also felt at rest. Signs and symptomsof peripheral vascular
disease include the following:

 decreased or absent distal pulses


 hair loss
 thickened nails
 shiny skin
 a skeletonized appearance
 pallor on elevation;
 rubor on dependency
 ulcers and gangrene

Finally, the progression of the disease is closely associated with cigarette smoking and diabetes mellitus.
Diabetic control and smoking cessation slows the progression of the disease.

Lymphatic obstruction (choice A) is a blockage of the lymph nodes and their vessels that drain fluid from
tissues throughout the body and allow immune cells to travel where they are needed. Lymphatic
obstruction is also called lymphedema, which means swelling of the lymph passages. Lymph nodes may be
enlarged for any reason. The main symptom is persistent (chronic) swelling, usually of the arm or leg.

Arteriosclerosis obliterans (answer B) is typically seen in medium and large arteries of the lower extremity

25
and characterized by fibrosis of the intima and calcification of the media. The lumen of the vessel may be
obliterated or markedly narrowed. It is often associated with hypertension and symptoms of Peripheral
artery disease appear.

Medial calcific sclerosis (choice D) is a form of arteriosclerosis or vessel hardening, where calcium deposits
form in the middle layer of the walls of medium sized vessels. It does not actually cause narrowing of the
vessel lumen. In advanced cases, vessels may become rigid and lose their distensibility. It is seen mostly
in the elderly and diabetics, and commonly involves the arteries of the thyroid and uterus.

Venous thrombosis (option E) is the development of a blood clot (thrombus) in a vein due to reduced blood
flow and abnormal coagulation. It usually begins with injury to the vein. The process may occur in
superficial or deep veins. DVT's produces no symptoms in 50% of individuals; the other 50% may complain
of swelling, tenderness, pain, skin discoloration and warmth in the affected area.

A 34 year old black girl has her right breast removed because of a large mass that has been growing for
over 10 years. The tumor weighs 1400g and is found to have a bulging, very firm, lobulated surface with a
whorl like pattern.

This neoplasm is most likely which of the following?

cystosarcoma phylloides
intraductal carcinoma
Duct ectasia
fibroadenoma
Granulomatous mastitis

Explanation

The correct answer is choice D

Fibroadenomas of the breast are small, solid, rubbery, noncancerous, harmless lumps composed of fibrous
and glandular tissue. Unlike typical lumps from breast cancer, fibroadenomas are easy to move, with
clearly defined edges. A fibroadenoma is usually diagnosed through clinical examination, ultrasound or
mammography, and often a needle biopsy sample of the lump

Fibroadenomas arise in the terminal duct lobular unit of the breast. They are the most common breast
tumor in adolescent women. They also occur in a small number of post-menopausal women. Their
incidence declines with increasing age, and they generally appear before the age of thirty years, probably
partly as a result of normal estrogenic hormonal fluctuation. Although fibroadenoma is considered a
neoplasm, some authors believe fibroadenoma arises from hyperplasia of normal breast lobule
components.

The typical case is the presence of a painless, firm, solitary, mobile, slowly growing lump in the breast of a
woman of childbearing years. In the male breast, fibroepithelial tumors are very rare, and are mostly
Phyllodes tumors.

Most fibroadenomas are left in situ and monitored by a doctor, or the patient in question. Some are treated
by surgical excision. They are removed with a small margin of normal breast tissue if the preoperative
clinical investigations are suggestive of the diagnosis.

Because needle biopsy is often a reliable diagnostic investigation, some doctors may decide not to operate

26
to remove the lesion, and instead opt for clinical follow-up to serially observe the lesion over time using
clinical examination and mammography to determine the rate of growth, if any, of the lesion. A growth
rate of less than sixteen percent per month in women under fifty years of age, and a growth rate of less
than thirteen percent per month in women over fifty years of age have been published as safe growth rates
for continued non-operative treatment and clinical observation. Fibroadenomas have not been shown to
recur following complete excision or transform into phyllodes tumours following partial or incomplete
excision.

cystosarcoma phylloides (choice A) is typically a large, fast growing mass that forms from the periductal
stromal cells of the breast. They account for less than 1% of all breast neoplasms. This is predominantly a
tumor of adult women, with very few examples reported in adolescents. Patients typically present with a
firm, palpable mass. These tumors are very fast growing, and can increase in size in just a few weeks.
Occurrence is most common between the ages of 40 and 50, prior to menopause. This is about 15 years
older than the typical age of patients with fibroadenoma, a condition with which Phyllodes tumors may be
confused.

intraductal carcinoma (choice B) is the most common type of breast cancer in women. It comes in two
forms: invasive ductal carcinoma(IDC), an infiltrating, malignant and abnormal proliferation of neoplastic
cells in the breast tissue, or ductal carcinoma in situ (DCIS), a noninvasive, possibly malignant, neoplasm
that is still confined to the milk ducts (lactiferous ducts), where breast cancer most often originates. On
physical examination, this lump usually feels much harder or firmer than benign breast lesions such as
fibroadenoma.

Duct ectasia (choice C) is a condition in which there is an obstruction of the lactiferous duct. Mammary
duct ectasia can mimic breast cancer. It is a disorder of premenopausal age. Signs of duct ectasia can
include nipple retraction, inversion, pain, and sometimes bloody discharge. Histologically, dilation of the
large duct is prominent. Pathogenesis may be a reaction to stagnant colostrum.

Granulomatous mastitis (choice E) are multinucleated giant cells and epithelioid histiocytes around lobules.
Often minor ductal and periductal inflammation is present. The lesion is in some cases very difficult to
distinguish from breast cancer. Patients usually present with a distinct firm mass mostly in the subareolar
region. It occurs on average 2 years and almost exclusively up to 6 years after pregnancy, usual age range
is 17 to 42 years. Use of hormonal contraceptives, prolactin raising medications and hyperprolactinemia
have been implicated in the pathogenesis or as predisposing factors.

A 30 year old man with bright red blood per rectum (BRBPR) with bowel movements presents to the
emergency room after experiencing an episode of syncope. He has no significant medical and his family
history is noncontributory. He also complains of some constipation but generally no change in bowel
movements, weight loss, or abdominal pain. He denies black tarry stools or hematochezia but his stools
are guaiac positive. His vitals signs are stable and the rest of the exam is normal.

What is the next appropriate step in management after IV access is established?

Colonoscopy
Colonoscopy and banding of hemorrhoids
Sigmoidoscopy and banding of hemorrhoids
Reassurance
Surgical intervention

Explanation

The correct answer is choice C

27
Lower gastrointestinal bleedin (LGIB), refers to any form of bleeding in the lower gastrointestinal tract.
LGIB is a common ailment seen at emergency departments. Approximately 85% of Lower gastrointestinal
bleeding involves the colon, 10% are from bleeds that are actually Upper gastrointestinal bleeds, and 3 -
5% involve the small intestines. A lower Gastrointestinal Bleed is referred as any bleed that occures distal
to the ligament of Treitz and superior to the anus. This includes the last 1/4 of the duodenum and the
entire area of the jejunum, ileum, colon, rectum, and anus.

The stool of a person with a lower gastrointestinal bleed is a good not infallible indication of where the
bleeding is occurring. Black tarry appearing stools medically referred to as melena usually indicates blood
that has been in the GI tract for at least 8 hours. Melena is four-times more likely to come from an upper
gastrointestinal bleed than from the lower GI tract; however, it can also occur in either the duodenum and
jejunum, and occasionally the portions of the small intestine and proximal colon. Bright red stool, called
hematochezia, is the sign of a fast moving active GI bleed. The bright red or maroon color is due to the
short time taken from the site of the bleed and the exiting at the anus. The presence of hematochezia is
six-times greater in a LGIB than with a UGIB,

Occasionally, a person with a LGIB will not present with any signs of internal bleeding. In these cases, a
diagnostic assessment or pre-assessment should watch for other signs and symptoms that the patient may
present with. These include, but are not limited to, hypotension, tachycardia, angina, syncope, weakness,
confusion, stroke, myocardial infarction, shock, and heart attack.

Evaluations will most often be conducted by either a clinic triage nurse, emergency department nurse,
and/or a physician or other clinican. The initial assessment will include the appearance of the individual,
their vital signs, and mental status. A patient history will help reveal a disposition or history of LGIBs or
potential differential diagnosis.

Orthostatic vital signs are often used as a indicator of hypovolemia. Laboratory test will also help give
indications of a LGIB. Hemoglobin, hematocrit, and platelets are very good physical signs of hypovolemia
or blood loss amenia. Aspiration of the stomach contents by way of a nasogastric tube (NG tube) will help
differentiate between either a UGIB and a LGIB. A negative presence of blood will help to rule out an UGIB.
IIf a patient is suspected of having severe blood loss they will most likely be placed on a vital sign monitor
and administered oxygen either by nasal cannula or simple face mask. An intravenous catheter will be
placed into an easily accessible area and IV fluids will be administered to replace lost volume.

Although the differential diagnosis for LGIB is extensive, this patient most likely has bleeding hemorrhoids
that can be visualized by sigmoidoscopy and treated by banding. A sigmoidoscopy only examines up to the
sigmoid, the most distal part of the colon, while colonoscopy examines the whole large bowel. Hemorrhoids
rarely extend past the distal colon so a colonscopy is unnecessary.

Full colonoscopy (choice A) is not necessary for symptomatic bleeding hemorrhoids. Colonscopies are not
completely benign procedures and carry the risk of gastrointestinal perforation. The risk of perforation
roughly doubles with colonscopies compared to sigmoidoscopies. It is also inappropriate to not treat a
symptomatic bleeding hemorrhoid.

Full colonoscopy and banding of hemorrhoids if present (choice B) is not appropriate because the benefit of
visualizing the colon past the anorectal junction is not outweighed by the risk of perforation for bleeding
hemorrhoids.

Reassurance (choice D) is inappropriate for symptomatic bleeding hemorrhoids. There are a number of
therapies that can be undertaken to stop bleeding hemorrhoids including rubber band ligation and
scleropterapy. Rubber band ligation is a procedure in which elastic bands are applied onto an internal
hemorrhoid at least 1 cm above the dentate line to cut off its blood supply. Within 5–7 days, the withered
hemorrhoid falls off. The cure rate has been found to be about 87%. Sclerotherapy involves the injection of
a sclerosing agent (such as phenol) into the hemorrhoid. This causes the vein walls to collapse and the
hemorrhoids to shrivel up. The success rate at four years is 70%.

Surgical intervention (choice E) for bleeding hemorrhoids is not warranted for a stable patient. If a patient
is suspected of having severe blood loss they will most likely be placed on a vital sign monitor and
administered oxygen either by nasal cannula or simple face mask. An intravenous catheter will be placed

28
into an easily accessible area and IV fluids will be administered to replace lost volume. Surgical
intervention is warranted in some cases. It is most likely that a sugical consult will be ordered if the patient
is unable to be stabilized by non-invasive techniques, or a perforation is found that requires surgery.

On postoperative day three for a coronary artery bypass, a 56 year old male develops a pounding heart
and shortness of breath. He feels that he is going to pass out but denies having chest pains. His BP is
120/80 with a pulse of 210. An EKG is performed as shown below. Attempts are made to lower his
tachycardia with carotid neck massage and splashing cold water on his face, but the attempts fail.

What is the next appropriate step in management?

Metoprolol
Verapamil
Cardioconversion
Adenosine
Amidoarone

Explanation

The correct answer is choice D

Tachycardias may be classified as either narrow complex tachycardias (supraventricular tachycardias) or


wide complex tachycardias. "Narrow" and "wide" refer to the width of the QRS complex on the ECG.
Narrow complex tachycardias tend to originate in the atria, while wide complex tachycardias tend to
originate in the ventricles. Tachycardias can be further classified as either regular or irregular.
Supraventricular tachycardia (SVT), which is a tachycardia paced from the atria or the AV node. SVT
rhythms include: Atrial fibrillation, AV nodal reentrant tachycardia, AV reentrant tachycardia, Junctional
tachycardia.

Symptoms can come on suddenly and may go away without treatment. They are caused for a reason
other than stress, exercise, or emotion. They can last a few minutes or as long as 1 or 2 days, sometimes
continuing until treated. The rapid beating of the heart during SVT can make the heart a less-effective
pump, decreasing cardiac output and blood pressure. The following symptoms are typical with a rapid
pulse of 150–251 or more beats per minute: pounding heart, shortness of breath, chest pain, rapid
breathing, dizziness, and loss of conciousness.

In general, SVT is not life threatening, but episodes should be treated or prevented. The SVTs can be
separated into two groups, based on whether they involve the AV node for impulse maintenance or not.
Those that involve the AV node can be terminated by slowing conduction through the AV node. Those
that do not involve the AV node will not usually be stopped by AV nodal blocking maneuvers. These
maneuvers are still useful however, as transient AV block will often unmask the underlying rhythm
abnormality.

In the case of narrow complex tachycardias (junctional, atrial or paroxysmal), the treatment in general is
to first give the patient adenosine (to slow conduction through the AV node) and then perform Valsalva
maneuvers to slow the rhythm. If this does not convert the patient, amiodarone, calcium channel
blockers or beta-blockers are commonly employed to stabilize the patient. Again as in atrial fibrillation, if
a patient is unstable, the decision to electrically cardiovert him/her should be made.

Metoprolol (choice A) iis a selective β1 receptor blocker that is used for rate control in supraventricular
tachycardia. Metoprolol is used for followup therapy after adenosine has aborted the supraventricular

29
tachycardia.

Verapamil (choice B) is an L-type calcium channel blocker. Since calcium channels are especially
concentrated in the sinoatrial and atrio-ventricular nodes, these agents can be used to decrease impulse
conduction through the AV node, thus protecting the ventricles from atrial tachyarrhythmias. Verapamil is
used for followup therapy after adenosine has aborted the supraventricular tachycardia.

cardioconversion (choice C) is a medical procedure by which an abnormally fast heart rate or cardiac
arrhythmia is converted to a normal rhythm, using electricity or drugs. Synchronized electrical
cardioversion uses a therapeutic dose of electric current to the heart, at a specific moment in the cardiac
cycle. Pharmacologic cardioversion, also called chemical cardioversion, uses antiarrhythmia medication
instead of an electrical shock. If the patient is stable, adenosine may be administered first, as the
medicine performs a sort of "chemical cardioversion" and may stabilize the heart and let it resume normal
function on its own without using electricity. Synchronized electrical cardioversion is used to treat
hemodynamically significant supraventricular (or narrow complex) tachycardias,

amidoarone (choice E) is an antiarrhythmic agentused for various types of tachyarrhythmias, both


ventricular and supraventricular (atrial) arrhythmias that do not involve the AV node. Amiodarone is
categorized as a class III antiarrhythmic agent, and prolongs phase 3 of the cardiac action potential.
Amiodarone shows beta blocker-like and potassium channel blocker-like actions on the SA and AV nodes,
increases the refractory period via sodium- and potassium-channel effects, and slows intra-cardiac
conduction of the cardiac action potential, via sodium-channel effects.

A six week old male infant presents with a history of projectile non bilous vomitting that occurs shortly after brest feeding.

The treatment of choice for the suspected condition includes which of the following:

vagotomy
pyloroplasty
pyloromyotomy
Heller's myotomy
Nissen fundoplication

Explanation

The correct answer is Choice C

Idiopathic hypertrophic pyloric stenosis (IHPS) is a condition characterized by a narrowing of the pylorus,
which connects the stomach to the duodenum. There is hypertrophy of the circular muscles at the gastro-
duodenal junction, leading to a reduction in the size of the lumen and obstruction of gastric emptying. The
incidence is about 3/1000 live births and the cause is unknown. It is commoner in the first-born male
children and has a higher incidence in Caucasians compared to the Black race.

Clinically, IHPS presents in neonates with initial non-bilious vomiting at about the 4th -8th week of life.
Vomiting typically occurs shortly after every feeding, with increasing intensity which eventually becomes
projectile. The baby usually appears hungry and sucks vigorously after every episode of vomiting.

Late presentations and delayed diagnosis gives a clinical picture of dehydration and shock in an infant with
malnutrition and evidence of poor weight gain.

30
On physical examination, an enlarged pylorus may be felt in the right upper quadrant, just below the costal
margin. This mass is typically described as an “olive”. The patient must be calm and co-operative for the
examiner to appreciate this finding.

Ultrasound diagnosis is the standard imaging investigation. Barium studies and upper GI endoscopy also
provide useful information to assist in management of the condition.

The definitive treatment is surgical repair of the stenosis. Pyloromyotomy (Choice C) is the recommended
surgical operation as it gives excellent results. Ramstedt pyloromyotomy is the preferred approach, via a
transverse right upper quadrant incision that splits the rectus muscle and fascia.

Preoperative management is aimed at correcting dehydration and electrolyte imbalance. Post-operatively,


recovery is usually excellent, and the child may resume graded oral feeds within 8 hours following the
surgery. Discharge home can follow as soon as the child remains dehydrated and can tolerate oral feeds.

Pyloroplasty (Choice B) refers to the surgical widening of the pylorus to aid gastric content emptying.
Pyloroplasty is commonly performed with a truncal vagotomy (Choice A) as part of the treatment for peptic
ulcer disease. Vagotomy is a delicate procedure involving the severance of the trunks of the vagus nerve,
leading to reduced gastric acid secretion.

Nissen Fundoplication (Choice D) is the surgical procedure used to treat gastro-esophageal reflux disease
(GERD) and hiatus hernia. The fundus of the stomach is wrapped around the lower end of the oesophagus
to provide additional support, thereby preventing the reflux of acid in GERD and the recurrence of a hiatus
hernia.

Heller’s Myotomy (Choice E) is a surgical procedure that involves cutting of the muscles of the lower
oesophageal or cardiac sphincters, allowing passage of food from the oesophagus into the stomach. It is
used in the treatment of Achalasia.

A neonate presented to the hospital with repeated bile-stained vomiting, abdominal distension, and failure
to pass meconium. Chest examination revealed associated bronchitis. The diagnosis was made as
suspected meconium ileus.

Regarding meconium ileus, all of the following statements are true except:

31
Is a rare cause of neonatal intestinal obstruction
many cases are associated with cystic fibrosis
Obstruction usually occurs in the distal ileum
Presents with neonatal bile-stained vomiting and
abdominal distension
A plain x-ray may show an intra-luminal 'soap bubble'
appearance

Click on image to Zoom

Explanation

The correct answer is choice A.

Meconium ileus accounts for approximately 33% of all cases of neonatal intestinal obstruction. Other
common causes of neonatal intestinal obstruction are jejuno-ileal atresia , dudenal extrinsic and intrinsic
obstruction.

Answer B is a correct phrase as meconium ileus is the early presentation of 20% of cases of cystic fibrosis.
Cystic fibrosis is an autosomal recessive disorder clinically characterized by the following triad:

 Chronic obstruction and infection of respiratory tract.


 Exocrine pancreatic insufficiency (lack of pancreatic lipase that liquifies the meconium).
 Elevated sweat chloride level.

Answer C is a correct phrase as usually at proximal part of ileum, the meconium is still fluidy to some
extent and then gradual insipissation occurs as it passes towards the terminal ileum.

Answer D is a correct phrase as meconium ileus causes intestinal obstruction which produces abdominal
distension and repeated vomiting that becomes bile-stained eventually.

Answer E is a correct phrase as the air become trapped inside the thick viscid meconium giving a
pathogonomonic soap bubble appearance.

A 24-year-old woman presents with severe diffuse abdominal pain. She has a six-month history of crampy
abdominal pain accompanied by intermittent bloody diarrhea. On physical exam she is found to have
marked abdominal distention. Further workup reveals toxic megacolon and she undergoes an emergent
colectomy. After the operation, she is brought to the SICU and the decision is made to feed her through
total parental nutrition (TPN).

Which one of the following constituents in your TPN bag is MOST likely to INCREASE stupor?

32
Glucose
Leucine
Pantothenic acid
L-Tryptophan
Lipids

Explanation

The correct answer is choice D

Parenteral nutrition (PN) is feeding a person intravenously, bypassing the usual process of eating and
digestion. The person receives nutritional formulas containing salts, glucose, amino acids, lipids and added
vitamins. It is called total parenteral nutrition (TPN) when no food is given by other routes.

Parenteral nutrition is provided when the gastrointestinal tract is nonfunctional because of an interruption
in its continuity or because its absorptive capacity is impaired. It has been used for comatose patients,
although enteral feeding is usually preferable, and less prone to complications. Indications: TPN may be
the only feasible option for patients who do not have a functioning GI tract or who have disorders requiring
complete bowel rest, such as the following: Crohn's disease or ulcerative colitis, bowel obstruction, certain
pediatric GI disorders, e.g., congenital GI anomalies, prolonged diarrhea regardless of its cause, or short
bowel syndrome due to surgery.

The most common complication of PN use is bacterial infection, usually due to the increased infection risk
from having an indwelling central venous catheter. In patients with frequent bacterial infections, fungal
infections can also occur.

Total parenteral nutrition increases the risk of acute cholecystitis due to complete unusage of
gastrointestinal tract, which may result in bile stasis in the gallbladder. Other potential hepatobiliary
dysfunctions include steatosis, steatohepatitis, cholestasis, and cholelithiasis. Such complications are
suggested to be the main reason for mortality in people requiring long-term total parenteral nutrition, such
as in short bowel syndrome.

Tryptophan is one of the 20 standard amino acids, as well as an essential amino acid in the human diet. In
addition, tryptophan functions as a biochemical precursor for serotonin and niacin. Once inside the CNS,
tryptophan is converted into serotonin in the raphe nuclei by the normal enzymatic pathway. The resultant
serotonin is further metabolised into melatonin by the pineal gland. Hence, tryptophan causes drowsiness
by sleep-promoting melatonin in the brain

Glucose (choice A) in TPN is known to cause liiver failure, often related to non-alcoholic steatohepatitis
(NASH). Liver dysfunction can be limited to a reversible cholestatic jaundice and to fatty infiltration
(demonstrated by elevated transaminases) by restricting glucose or adding insulin to the TPN mixture.
Severe hepatic dysfunction is a rare complication.

Leucine (choice B) causes delirium, neurologic compromise, and can be life-threatening if given in excess
in TPN.

Pantothenic acid (choice C) is a water-soluble vitamin. Pantothenic acid is used to synthesize coenzyme-A
(CoA), and well as to synthesize and metabolize proteins, carbohydrates, and fats.

Excess pantothenic acid is widely known to cause nausea, headaches, diarrhea and a lack of energy. The
lack of energy is believed to be the depleted vitamin B12 (cobalamin), as massive amounts of vitamin B5
will deplete other vitamin B components. However, it does not cause stupor.

Lipids (choice E) that are given in TPN infusions are known to cause venous thrombosis and rarely
priapism.

33
3 week old boy has had projectile vomiting and weight loss over the past week. You suspect pyloric
stenosis. Which of the following is/are correct regarding pyloric stenosis?

1. a mixed respiratory and metabolic alkalosis is the usual acid base disorder associated with this
condition
2. even if a pyloic tumor is palpable, roentgenographic studies of the upper gastrointestinal tract should
be obtained prior to surgery
3. a pyloric tumor is usually palpable in the left upper quadrant
4. ultrasonography is a reliable diagnostic procedure

1,2,3 only
1,3 only
2,4 only
4 only
All of the above are correct

Click on image to Zoom

Explanation

The correct answer is choice D.

Pyloric stenosis involves hypertrophy of the circular muscle of the pylorus, resulting in narrowing and
obstruction of the pyloric channel by compression of longitudinal folds of mucosa. Gastric outlet obstruction
results in emesis, which is characteristically nonbilious and projectile. Protracted emesis, as well as failure
of the stomach to empty into the duodenum, results in progressive dehydration, electrolyte abnormalities,
acid-base disorders, weight loss, and, potentially, shock.

Phrase 4 only is true as the ultrasonography can show the following criteria which are reliable and specific
for pyloric stenosis :

 Muscle thickness (serosa to mucosa) greater than 3 mm.


 Target sign on transverse images of the pylorus.(See the picture)
 Pyloric channel length greater than 17 mm.
 Pyloric thickness (serosa to serosa) of 15 mm or greater.
 Failure of the channel to open during a minimum of 15 minutes of scanning.
 Retrograde or hyperperistaltic contractions.
 Antral nipple sign (a prolapse of redundant mucosa into the antrum, which creates a pseudomass)

Phrase 1 is false as there is only metabolic alkalosis in such case. The alkalosis is caused by loss H and Cl

34
ions due to repeated vomiting. There is no respiratory element in the development of the alkalosis.

Phrase 2 is false as once a clear history is obtained and the pyloric tumor is palpated, the diagnosis is
established and there would be no need for further imaging studies.

Phrase 3 is false as the pyloric tumor is located in the epigastrium just above the umbilicus, either in the
midline or just to the right.

A 45-year-old woman with no significant past medical history presents with a firm 3-cm mass in the right
thyroid lobe with no clinical evidence of nodal involvement. She has no family history of cancers. A fine-
needle aspiration (FNA) is performed and histologic examination confirms a diagnosis of papillary cancer.

What is the next appropriate step in management?

Reassure and Follow up in 1-Year


Lobectomy and isthmusectomy
Radioiodine ablation
Total thyroidectomy
Ultrasound of thyroid

Explanation

The correct answer is choice D

Papillary thyroid cancer or papillary thyroid carcinoma is the most common type of thyroid cancer,
representing 75% to 85% of all thyroid cancer cases. It occurs more frequently in women and presents in
the 30-40 year age group. It is also the predominant cancer type in children with thyroid cancer, and in
patients with thyroid cancer who have had previous radiation to the head and neck. It has the most
favorable prognosis of all thyroid cancers.

Most often the first symptom of thyroid cancer is a nodule in the thyroid region of the neck. However,
many adults have small nodules in their thyroids, but typically under 5% of these nodules are found to be
malignant. Sometimes the first sign is an enlarged lymph node. Later symptoms that can be present are
pain in the anterior region of the neck and changes in voice.

Thyroid cancer is usually found in a euthyroid patient, but symptoms of hyperthyroidism or hypothyroidism
may be associated with a large or metastatic well-differentiated tumor. Thyroid nodules are of particular
concern when they are found in those under the age of 20. The presentation of benign nodules at this age
is less likely, and thus the potential for malignancy is far greater.

Thyroid cancer may require surgery. Common surgeries include thyroidectomy, lobectomy, and
tracheostomy. Surgical treatment is dependent on the size of the disease:

 Minimal disease (diameter up to 1.0 centimeters) - hemithyroidectomy (or unilateral lobectomy)


and isthmectomy may be sufficient.
 Gross disease (diameter over 1.0 centimeters) - total thyroidectomy, and central compartment
lymph node removal is the therapy of choice. Additional lateral neck nodes can be removed at the
same time if an ultrasound guided FNA and thyrobulin TG cancer washing was positive on the pre-
operative neck node ultrasound evaluation.

35
Lobectomy and isthmusectomy (choice B) is indicated for minimal disease up to 1.0 cm in diamater.

Radioiodine ablation (choice C) may be used when the cancer is unresectable or recurrence after resection.
Radioactive Iodine-131 is used in patients with papillary or follicular thyroid cancer for ablation of residual
thyroid tissue after surgery and for the treatment of thyroid cancer. It may also be used to relieve pain
from bone metastasis.

Ultrasound of thyroid (choice E) is performed to confirm the presence of a nodule and assess the status of
the whole gland. It is not performed after a fine needle aspiration confirms the presence of thyroid
neoplasm.

A 3 week old baby is brought to the emergency department with a 36 hour history of progressive
vomiting. The birth weight was 3.1 kg. On physical examination, his weight is 2.8 kg; he has at least 5%
dehydration to date and is very irritable but not toxic. His blood work reveals:

 pH: 750
 paCO2: 30 mm/Hg
 HCO3: 31 mmol/L
 Base excess; 7 mmol/L
 Potassium: 3.0 mmol/L
 Sodium: 135 mmol/L
 Chloride: 70 mmol/L
 Urea nitrogen 7.2 mmol/L

Which one of the following diagnoses is the most likely?

obstructive uropathy
gastroenteritis
pyloric stenosis
congenital adrenal hyperplasia
cystic fibrosis

Explanation

The correct answer is choice C.

Pyloric stenosis involves hypertrophy of the circular muscle of the pylorus, resulting in narrowing and
obstruction of the pyloric channel by compression of longitudinal folds of mucosa. Gastric outlet obstruction
results in emesis, which is characteristically nonbilious and projectile. Protracted emesis, as well as failure
of the stomach to empty into the duodenum, results in progressive dehydration (explains the weight loss),
electrolyte abnormalities, acid-base disorders, weight loss, and, potentially, shock.

Gastric fluid loss is associated with the loss of H and Cl ions (diminished blood Cl) leading to metabolic
alkalosis (elevated pH, diminished PaCO2, increased HCO3 and increased base excess). Urinary Na and
HCO3 losses, which compensate for Cl losses, increase this alkalosis. However, urinary excretion of K and
H increases in an attempt to preserve Na and blood volume (normal Na level and diminished K level)

Answer A, obstructive uropathy, is false as the renal function is normal (normal BUN). Also there would be
fluid retention leading to weight gain and not weight loss. Obstructive uropathy cannot also explain the

36
repeated vomiting.

Answer B is false as gastroenteritis would not produce such electrolyte disturbances unless in severe cases.
Also the patient doesn't’ look toxic.

Answer D, congenital adrenal hyperplasia, is false as in this disease there would be hyponatremia and
hyperkalemia.

Answer E is false as cystic fibrosis is not usually presented with such electrolyte disturbances. Also it is
usually presented with respiratory distress along with manifestations of intestinal obstruction.

A 5 year old boy has a midline swelling of the neck that moves with deglutition and is located below the hyoid bone.

The mass is likely to be which of the following?

Branchial cleft cyst


Cystic hygroma (lymphatic malformation)
Teratoma (dermoid cyst)
Thyroglossal duct cyst
Midline Thyroid cyst

Explanation

The correrct answer is Choice D

Thyroglossal duct cyst (TGDC) is the most common midline neck mass usually located at or below the level
of the hyoid bone. It is formed from remnants of the thyroglossal duct which may persist after birth. The
thyroglossal duct is the hollow canal through which the developing thyroid gland descends from its origin in
the foramen caecum at the base of the tongue to its location in the anterior triangle of the neck. The duct
usually involutes after birth; however, if this fails to occur, the tract may remain dormant for years until
some stimulus causes cystic dilatation. TGDC is usually located at or below the level of the hyoid bone. It
can occur at any age, but it is most common before 20 years of age.

Clinically, it presents as a cystic painless mid-line neck mass in the region of the hyoid bone. It becomes
symptomatic if inflamed, resulting in pain and swelling. Because of its connection with the hyoid bone and
the base of the tongue, TGDC usually move on swallowing and protrusion of the tongue.

Antibiotics may be indicated if the cyst becomes infected. Definitive treatment is complete surgical
excision. Sistrunk’s procedure is the recommended surgical approach. The operation includes excision of
the cyst in continuity with the mid portion of the body of the hyoid bone and a small block of muscle
around the foramen caecum.

Branchial cleft cyst (Choice A) is an uncommon neck swelling found along the anterior border of the
sternocleidomastoid, usually at the junction of its upper and middle third. It is formed from remnants of
the second branchial cleft which persist after embryogenesis. It is found in young adults and presents as a
fluctuant mass that doesn’t transilluminate. It does not move on swallowing. Definitive treatment is by
surgical excision. The cyst may become secondarily infected during an upper respiratory tract infection. In

37
this case, excision should be delayed to avoid creating a branchial fistula. Care must be taken not to
damage the carotid vessels and internal jugular vein which usually lie deep to the swelling.

Cystic hygroma (Choice B) results from congenital malformation of the lymphatic system. It usually presents in early
childhood as a painless soft multi-locular cystic swellings. The swelling is commonly found in the neck and axilla, although
it can occur anywhere. Small lesions require no treatment, but large cervical lesions may cause airway obstruction.
Surgical excision is the best option, but this can be difficult in the swelling infiltrates surrounding tissue. Use of sclerosants
e.g. lyophilized product of Streptococcus pyogens, is an option of treatment.

A teratoma (Choice C) is a germ cell tumour derived from pluripotential stem cells. The tumour usually
contains elements of different tissue derived from all three germ layers. The common sites for teratomas
are the ovary and testis. In infancy, the commonest site is the sacrococcygeus. A rare form of head and
neck teratomas arises primarily from remnants of Rathke's pouch in the sphenoid bone region, the tongue
and lateral part of the neck. Treatment is by surgical careful surgical excision as recurrence is very
common.

Thyroid cyst (Choice E) is a fluid filled sac in the thyroid gland. The diagnosis is made clinically by
observation and palpation. Confirmation is usually by ultrasound scan. Usually, thyroid cysts are drained by
fine needle aspiration, and the fluid sent for cellular analysis to rule out any occult malignancy.

A 62 year old healthy white male presents with a chief complaint of difficulty reading for the past two
weeks. He only wears spectacle correction for near tasks. He went to Walmart OTC and obtained a new
pair of glasses w/o relief of his symptoms. The onset was abrupt, painless and he denies headache, or
other neurologic symptoms. He notes some nausea since the occurrence. Medical history is significant for
mild hypertension and he is using lisinopril daily.

Relevant vital signs:

 BP 140/100 mm hg
 Glucose serum level 225 mg% (50-100)
 Visual acuity OD 20/25
 Visual acuity OS 20/30
 IOP OD 16
 IOP OS 16 mm hg (Goldman tonometry)

The patient’s appearance is shown in the accompanying photograph.

Which of the following is the correct answer:

Sixth Nerve Palsy


Fourth Nerve Palsy OS
Fourth Nerve Palsy OD
Third Nerve Palsy OD
Medial longitudinal fascicle (MLF) Syndrome

Click on image to Zoom

38
Explanation

The correct answer is choice C

The patient noted diploplia when reading because the superior oblique muscle is most active in
convergence and infragaze…the reading position. He presents with a left head tilt to minimize the effect of
a superior oblique palsy in the right eye. In this case, decreased in torsion of the RSO causes the patient to
tilt his head in the direction of the muscles action, thereby minimizing or eliminating the symptom.

Typical etiologies are most often traumatic, vascular, tumors rarely, and are often idiopathic with a
negative workup.

Treatment consists of weakening the agonist muscle, the inferior oblique by recessing it. Surgery for
superior oblique palsy is often only partially successful and the patient must be fitted with spectacle prisms
to eliminate the diploplia.

Choice A is incorrect because the 6th nerve (abducens) controls lateral abduction and would create a head
turn, not a head tilt. The affected eye would turn in (esotropia) in the primary position, causing horizontal
diploplia.

Choice B is incorrect because the head tilt would be in the opposite direction, (right head tilt) and the right
eye would be deviated superiorly.

Choice D is unlikely because it would affect all muscles except the superior oblique and lateral rectus
muscles. There would also be a ptosis since the levator palpebra muscle is also innervated by the 3rd
nerve (oculomotor).

Medial longitudinal fascicle (MLF) forms the ventral border of the occulomotor and tochlear nucleus in the
pons. MLF serves an important function in coordinating the movement of the eye in vision by
interconnecting the occulomotor nuclei, trochlear nuclei and abducens nuclei. Bilateral (unilateral much less
common) internuclear ophthalmoplegia, so called MLF syndrome (choice E), is a classic, and often the
initial, manifestation of multiple sclerosis. It is caused by demyelination in MLF. Manifestations of MLF
syndrome are featured by weakness in adduction of the ipsilateral eye with nystagmus on abduction of the
contralateral eye, and incomplete or slow abduction of the ipsilateral eye upon lateral gaze.

A 63-year-old man has had insulin dependent diabetes mellitus for over two decades. The degree of
control of his disease is characterized by the laboratory finding of a hemoglobin A1C of 10.1%. He has
noted episodes of abdominal pain following meals for the past few weeks. Postprandial pain, typically
starts 20-30 minutes after his last meal, that may last up to 60-90 minutes. These episodes have
worsened over the past year. There is no history of nausea or vomitting.

On physical examination, you have a well appearing obese male that is in no distress. Cardiovascular and
respiratory exam are normal. On abdominal examination, there are no masses and no organomegaly, and
he has no tenderness to palpation. Bowel sound are normal in all 4 quadrants.

Which of the following pathologic findings is most likely to be present in this man?

39
Chronic pancreatitis

Gastric carcinoma
Mesenteric artery occlusion
Diabetic gastroparesis
Gastric ulcer

Explanation

The correct answer is choice C

Chronic abdominal pain (CAP) in patients with diabetes may be due to chronic mesenteric ischemia (CMI).
All above options are differentials of CAP that occur after a meal.

Atherosclerosis is 2–4 times more common in patients with diabetes and affects mainly carotid, coronary,
iliac and lower limb arteries as well as the aorta. Another less common complication is chronic mesenteric
ischemia (CMI, intestinal angina), caused by atherosclerotic obstruction of the celiac artery and its
branches and results in episodic or constant intestinal hypoperfusion.

During meals, as the demand for blood in the intestinal tract rises, the vessels are not able to supply this
increased demand, and angina-like pain results.The typical presentation is crampy postprandial abdominal
pain between 10 minutes and 3 hours after a meal and as a result, they are afraid to eat (phagophobia).
This leads to anorexia and weight loss over time. Later changes in bowel habits occur, many report
diarrhea or constipation. They may also complain of frank blood in their stools as their bowel begins to
develop necrosis, and occult testing of stool may return positive results because of the sloughing of dead
ischemic bowel.

At least two of the three main splanchnic arteries must be significantly occluded in order for CMI to be
symptomatic. Abdominal examination findings may be benign. Patients with a history of chronic
mesenteric ischemia may have physical findings consistent with a malnourished state. Diagnostic
procedure of choice is conventional angiography. Laboratory examination should include the following:

 prothrombin time (PT)


 activated partial thromboplastin time (aPTT)
 complete blood count (CBC), which may reveal leukocytosis and/or hemoconcentration
 chemistries, which may show acidosis or increased amylase or lactate dehydrogenase (LDH) levels
 chest radiography
 electrocardiography

In terms of treatment, revascularization of the occluded arteries is most often recommended. Risk factors
for arterial thrombosis include atherosclerosis, hypovolemia, congestive heart failure, recent MI, advanced
age, and intra-abdominal malignancy. a significantly higher risk of acute mesenteric ischemia in the
patients with IBD.

Most people with chronic pancreatitis (Choice A) experience upper abdominal pain, although some people
have no pain at all. The pain may spread to the back, feel worse when eating or drinking, and become
constant and disabling. In some cases, abdominal pain goes away as the condition worsens, most likely
because the pancreas is no longer making digestive enzymes. Other symptoms
include nausea, vomiting, weight loss, diarrhea and oily stools. People with chronic pancreatitis often lose
weight, even when their appetite and eating habits are normal. The weight loss occurs because the body
does not secrete enough pancreatic enzymes to digest food, so nutrients are not absorbed normally. Poor
digestion leads to malnutrition due to excretion of fat in the stool.

40
The most worrisome diagnosis in someone who complains of postprandial pain is gastric cancer (choice B).
Stomach cancer is often asymptomatic or causes only nonspecific symptoms in its early stages. By the
time symptoms occur, the cancer has often reached an advanced stage (see below) which is one of the
main reasons for its poor prognosis.

Signs and symptoms of gastroparesis (choice D) include the following:

 heartburn
 pain in the upper abdomen
 nausea
 vomiting of undigested food—sometimes several hours after a meal
 early feeling of fullness after only a few bites of food
 weight loss due to poor absorption of nutrients or low calorie intake
 abdominal bloating
 high and low (variable) blood glucose levels
 lack of appetite
 gastroesophageal reflux
 spasms in the stomach area

Gastroparesis is the result of damage to the vagus nerve, which controls the movement of food through
the digestive system. Instead of moving through the digestive tract normally, the food is retained in the
stomach. Gastroparesis may occur in people with type 1 diabetes or type 2 diabetes. The vagus nerve
becomes damaged after years of high blood glucose, resulting in gastroparesis. In turn, gastroparesis
contributes to poor blood glucose control.

Given the clinical scenario, gastric ulcer (Choice E) is not likely to be the cause.

A healthy female infant is born via spontaneous vaginal delivery at 38 weeks gestation, weighing 3.5kg with an Apgar
score of 7 and 9 at one and five minutes respectively. Twenty four hours after birth, the mother is concerned that the infant
has failed to pass meconium. The infant is afebrile and breast feeding appropriately.

What is the appropriate initial management?

IVF and nasogastric tube aspiration


Defunctioning colostomy
Reassure mother and continue observation for another 24 hours
Posterior saggital anorectoplasty
Lateral Invertogram
Stool Enema or suppository

Explanation

The correct answer is choice C

The mother should be reassured as it is normal for passage of meconium to be delayed up to 48 hours
after birth. However, further delay will require an investigation of the infant. A few possible causes of

41
delayed passage of meconium include Imperforate anus, Hirschsprungs disease and Meconium Ileus.

Imperforate anus refers to a congenital anomaly resulting from failure of complete development and
canalization of the ano-rectal canal. It affects about 1 in 4000 new births; there is equal prevalence in
males and females. The clinical forms of imperforate anus include rectal atresia which may be high or low,
depending on the position of the deformity either above or below the levator ani muscle. Low lesions are
usually associated with rectovesical, recto-urethral or recto-vaginal fistulae. These associated anomalies
result in bladder and sexual dysfunction. Clinically, infants usually present with intestinal obstruction
manifesting as failure to pass meconium. The extent of the anal defect or rectal atresia can be confirmed
using a lateral invertogram (Choice E). The infant is inverted and the anus is marked with a radio-opaque
marker and a lateral x-ray taken. Air in the rectum will rise to the highest point of the rectum that is
patent indicating the extent of the atresia. Definitive treatment is surgical reconstruction. The preferred
method is the posterior saggital anorectoplasty (PSARP) (Choice D), usually for low defects. High defects
are usually managed initially with a colostomy (Choice B) and subsequent surgical intervention.

Meconium ileus is a clinical condition result in from inspissated meconium blocking the small bowel of
neonates. It is frequently associated with Cystic Fibrosis as 15% of patients with this condition present
with meconium ileus. Lack of pancreatic enzymes and loss of intestinal mucus leading to the formation of
thickened tenacious meconium which causes intraluminal obstruction in the terminal ileum. Infants with
meconium ileus usually present in the first few hours of life with abdominal distention, bile stained vomitus
and failure to pass meconium. Diagnosis is usually clinical, but can be aided by an abdominal radiograph
which shows a mottled soap-bubble appearance of meconium in the right iliac fossa due to the presence of
fat droplets. Treatment of the condition requires relieving of the intestinal obstruction using gastrograffin
enema or laparotomy in complicated cases. Concurrent management of CF is indicated if diagnosed.

Hirschsprungs disease is another congenital condition that presents with failure to pass meconium in the
first few days of life. It is caused by failure of ganglion cells of the myenteric and submucosal plexuses to
migrate into the hindgut, resulting in failure of coordinated peristalsis in the distal aganglionic segment of
the gut. 75% of cases are confined to the rectosigmoid colon (short segment disease), while 10 % of cases
involve the entire colon (long segment disease). Infants present with features of intestinal obstruction
abdominal detention, bile-stained vomiting and failure to pass meconium in the first few days of life.
Diagnosis is confirmed by suction rectal biopsy, which demonstrates the absence of ganglion cells. Anal
manometry and contrast studies may help determine the extent of gut involvement. Definitive
management is surgical, requiring an initial defunctioning colostomy (Choice B) and a subsequent pull-
through surgery, anastomosing the normal bowel to the anus.

A 3 months old infant was brought to you complaining of vomiting that started infrequently but now has
become projectile and recurrent after every feeding. On examination, the baby was malnourished and
vigorous peristalsis is seen in abdomen. An olive-like mass is palpated in upper right quadrant. The case
was diagnosed as hypertrophic pyloric stenosis.

Regarding infantile hypertrophic pyloric stenosis, which of the following statements is true:

Usually presents between 6 and 12 months of


age
The male:female ratio is 4:1
Has a strong familial predisposition
Pathologically shows hypertrophy of the
longitudinal muscle layer of the pylorus
Presents with bile-stained projectile vomiting

Click on image to Zoom

42
Explanation

The correct answer is C.

Hypertrophic pyloric stenosis is characterized by hypertrophy and hyperplasia of the circular muscle fibers
of the pylorus causing narrowing or even obstruction of the pyloric canal.

A familial predisposition is strongly suggested by the following :

 Occurs in 7% of infants of affected parents.


 Occurrence is reported in 3 successive generations of some families.
 85% coincidence in monozygotic twins.
 8% coincidence in dizygotic twins.

Answer A is a false phrase as 95% of cases are diagnosed within 3-12 months of age. Usually repeated
vomiting starts within 4 weeks of birth making the diagnosis most likely to occur very early.

Answer B is a false phrase as female : male ratio is 5 : 1. Highest incidence occurs in first-born male.

Answer D is a false phrase as the hypertrophy occurs in circular muscle layer not the longitudinal layer.

Answer E is a false phrase as the vomiting is characteristically non-bilious as the obstruction occurs prior to
the duodenum into which the bile is secreted.

A 46 year-old man complains of right upper quadrant pain and has jaundice on clinical exam. On review of
his labs, he has elevated alkaline phosphatase and increased direct bilirubin. An ultrasound scan of the
abdomen demonstrated a dilated common bile duct. He proceeds to endoscopic retrograde cholangio-
pancreatography (ERCP) for retrieval of stone from the common bile duct.

The staff doctor asks you where does the ampulla of Vater enter the bowel?

Jejunum
Ascending (fourth part) duodenum
Descending (second part) duodenum
Inferior (third part) duodenum
terminal ileum

Explanation

The correct answer is choice C

The common bile duct (ductus choledochus) is a tube-like anatomic structure in the human gastrointestinal
tract. It is formed by the union of the common hepatic duct and the cystic duct (from the gall bladder). It
is later joined by the pancreatic duct to form the ampulla of Vater. The ampulla of Vater, also known as the
hepatopancreatic ampulla, is specifically located at the major duodenal papilla. The Ampulla of Vater is an
important landmark, halfway along the second part of the duodenum, that marks the anatomical transition
from foregut to midgut

The two ducts are surrounded by the muscular sphincter of Oddi. When the sphincter of Oddi is closed,

43
newly synthesized bile from the liver is forced into storage in the gall bladder. When open, the stored and
concentrated bile exits into the duodenum. This conduction of bile is the main function of the common bile
duct. The hormone cholecystokinin, when stimulated by a fatty meal, promotes bile secretion by increased
production of hepatic bile, contraction of the gall bladder, and relaxation of the Sphincter of Oddi.

Several problems can arise within the common bile duct. While stones can frequently pass through the
common bile duct into the duodenum, some stones may be too large to pass through the CBD and may
cause an obstruction. One risk factor for this is duodenal diverticulum. If clogged by a gallstone, a
condition called choledocholithiasis can result. In this clogged state, the duct is especially vulnerable to an
infection called ascending cholangitis. Very rare deformities of the common bile duct are cystic dilations (4
cm), choledochoceles (cystic dilation of the ampula of Vater (3-8 cm)), and biliary atresia.

Choledocholithiasis causes jaundice and liver cell damage, and is a medical emergency. The diagnosis of
choledocholithiasis is suggested when the liver function blood test shows an elevation in bilirubin. The
diagnosis is confirmed either with an MRCP, ERCP, or an intraoperative cholangiogram. If the patient must
have the gallbladder removed for gallstones, the surgeon may choose proceed with the surgery, and obtain
a cholangiogram during the surgery. If the cholangiogram shows stone in the bile duct, the surgeon may
attempt to treat the problem by flushing the stone into the intestine or retrieve the stone back through the
cystic duct.

Jejunum (choice A) lies between the duodenum and the ileum. The change from the duodenum to the
jejunum is usually defined as the ligament of Treitz.

Ascending (fourth part) duodenum (choice B) reaches the inferior border of the body of the pancreas.
Then, it curves anteriorly and terminates at the duodenojejunal flexure where it joins the jejunum. The
duodenojejunal flexure is surrounded by a peritoneal fold containing muscle fibres: the ligament of Treitz.
The amuplla of vater does not drain into the ascending duodenum.

Inferior (third part) duodenum (choice D) begins at the inferior duodenal flexure and passes transversely
to the left, crossing the right ureter, right testicular/ovarian vessels, inferior vena cava, abdominal aorta,
superior mesenteric artery and the vertebral column. The amuplla of vater does not drain into the inferior
duodenum.

terminal ileum (choice E) is the most distal part of the small intestine. It connects to the cecum, the pouch
between the small and the large intestine, via the ileocecal valve. The amuplla of vater does not drain into
the terminal ileum.

You are called to examine a neonate who was born with excessive oral secretions. He has a sonorous
cough and on breast feeding he seems to choke. On examination, there were evident respiratory distress
and crepitations heard all around the chest. An orogastric tube is introduced but it was arrested. Also you
noticed that the baby has polydactyly and imperforate anus. Your initial diagnosis was esophageal atresia
with tracheo-esophageal fistula.

Regarding esophageal atresia and tracheo-oesophageal fistula (TOF), all of the following statements are
true except:

44
Most cases of esophageal atresia are associated with a
proximal TOF to the lower oesophagus

Radiological evidence of gas in the stomach confirms


the presence of distal TOF

Polyhydramnios is a common feature in a fetus with


oesophageal atresia

The diagnosis my be confirmed by the inability to pass a


nasogastric catheter

Oesophageal atresia with TOF is a part of Vactrel


syndrome

Click on image to Zoom

Explanation

The correct answer is choice A.

85% of cases are associated with distal TOF and only less than 1% of cases are associated with proximal
TOF. Tracheo-oesophageal fistula develops due to failure of fusion of tracheo-oesophageal folds. These
folds normally fuses at 4th to 6th week of gestation dividing the foregut into a ventral laryngo-tracheal
tube and a dorsal oesophageal tube. This leads to persistent abnormal communication between trachea
and oesophagus.

Choice B is a correct phrase as in presence of distal TOF, air passes through the fistula to stomach causing
gaseous distention.

Choice C is a correct phrase as polyhydramnios occurs in 33% of cases with distal TOF and 100% of cases
with absent TOF due to inability of the fetus to effectively swallow amniotic fluid.

Choice D is a correct phrase as on introducing orogastric tube it is arrested at the site of atresia
characteristically 10 to 11 cm from the lips.

Choice E is a correct phrase as VACTREL syndrome includes vertebral defects, anorectal malformation,
cardiovascular disease, TOF, renal anomalies and limb deformities.

A 34 year old lady presents with progressive dysphagia to both solids and liquids. Heartburn is not a
feature of her history. A barium swallow reveals a dilated esophagus with a hold up of barium and a rapid
taper at the cardia.

All of the following statements are true about this condition except?

45
incomplete LES relaxation
decreased LES tone
aperistalsis of the esophagus
most common form is primary achalasia
secondary to conditions such Chagas
disease

Click on image to Zoom

Explanation

The correct answer is choice B

Achalasia is an esophageal motility disorder involving the smooth muscle layer of the esophagus and the
lower esophageal sphincter (LES). It is characterized by incomplete LES relaxation, increased LES tone
(thus making choice B incorrect), and aperistalsis of the esophagus (inability of smooth muscle to move
food down the esophagus) in the absence of other explanations like cancer or fibrosis.

The main symptoms of achalasia are dysphagia (difficulty in swallowing) and regurgitation of undigested
food. Dysphagia tends to become progressively worse over time and to involve both fluids and solids.
Some achalasia patients also experience weight loss, coughing when lying in a horizontal position, and
chest pain which may be perceived as heartburn. The chest pain experienced, also known as cardio spasm
and Non Cardiac Chest Pain (NCCP) can often be mistaken for a heart attack. It can be extremely painful in
some sufferers. Food and liquid, including saliva, are retained in the esophagus and may be inhaled into
the lungs (aspiration).

The most common form is primary achalasia, which has no known underlying cause. It is due to the failure
of distal esophageal inhibitory neurons. However, a small proportion occurs secondary to other conditions,
such as esophageal cancer or Chagas disease (an infectious disease common in South America).

Due to the similarity of symptoms, achalasia can be mistaken for more common disorders such as
gastroesophageal reflux disease (GERD),hiatus hernia, and even psychosomatic disorders. Specific tests for
achalasia are barium swallow and esophageal manometry. In addition,endoscopy of the esophagus,
stomach and duodenum (esophagogastroduodenoscopy or EGD), with or without endoscopic ultrasound, is
typically performed to rule out the possibility of cancer. The internal tissue of the esophagus generally
appears normal in endoscopy, although a "pop" may be observed as the scope is passed through the non-
relaxing lower esophageal sphincter with some difficulty, and food debris may be found above the LES.

Various treatments are available, although none cure the condition. Certain medications or Botox may be
used in some cases, but more permanent relief is brought by esophageal dilatation and surgical cleaving of

46
the muscle (Heller myotomy).

A previously healthy 65 year old woman presents with jaundice that has developed in the past couple
months. She was recently diagnosed with diabetes a month ago. She denies abdominal pain but her stools
have been pale and her urine has been darker than usual. Upon examination, you see a jaundiced woman
with scleral icterus and a palpable gallbladder. Abdominal ultrasound reveals gall stones, a 1.7 cm
common bile duct, and a 2x3 cm mass in the head of the pancreas. Blood tests reveal elevated direct and
total bilirubin.

The most appropriate diagnostic test to perform next includes which of the following?

Endoscopic retrograde cholangiopancreatography (ERCP)


Selective angiography
Contrast CT scan of abdomen and pelvis
Percutaneous transhepatic cholangiography
Magnetic resonance cholangiopancreatography (MRCP)

Explanation

The correct answer is choice C

Pancreatic cancer is a malignant neoplasm of the pancreas. The prognosis is poor, with fewer than 5% of
those diagnosed still alive five years after diagnosis. Complete remission is still rare. About 95% of
exocrine pancreatic cancers are adenocarcinomas. The remaining 5% include adenosquamous carcinomas,
signet ring cell carcinomas, hepatoid carcinomas, colloid carcinomas, undifferentiated carcinomas, and
undifferentiated carcinomas with osteoclast-like giant cells. Exocrine pancreatic tumors are far more
common than pancreatic endocrine tumors, which make up about 1% of total cases

Pancreatic cancer is sometimes called a "silent killer" because early pancreatic cancer often does not cause
symptoms, and the later symptoms are usually nonspecific and varied. Therefore, pancreatic cancer is
often not diagnosed until it is advanced. Common symptoms include:

Pain in the upper abdomen that typically radiates to the back (seen in carcinoma of the body or tail of the
pancreas)

 Loss of appetite and/or nausea and vomiting


 Significant weight loss
 Painless jaundice (yellow skin/eyes, dark urine) when a cancer of the head of the pancreas (about
60% of cases) obstructs the common bile duct as it runs through the pancreas. This may also
cause pale-colored stool and steatorrhea.
 Trousseau sign, in which blood clots form spontaneously in the portal blood vessels, the deep
veins of the extremities, or the superficial veins anywhere on the body, is sometimes associated
with pancreatic cancer.
 Diabetes mellitus, or elevated blood sugar levels. Many patients with pancreatic cancer develop
diabetes months to even years before they are diagnosed with pancreatic cancer, suggesting new
onset diabetes in an elderly individual may be an early warning sign of pancreatic cancer.

The initial presentation varies according to location of the cancer. Malignancies in the pancreatic body or
tail usually present with pain and weight loss, while those in the head of the gland typically present with
steatorrhea, weight loss, and jaundice. The recent onset of atypical diabetes mellitus, a history of recent

47
but unexplained thrombophlebitis (Trousseau sign), or a previous attack of pancreatitis are sometimes
noted. Courvoisier sign defines the presence of jaundice and a painlessly distended gallbladder as strongly
indicative of pancreatic cancer, and may be used to distinguish pancreatic cancer from gallstones.
Tiredness, irritability and difficulty eating because of pain also exist. Pancreatic cancer is often discovered
during the course of the evaluation of aforementioned symptoms.

Liver function tests can show a combination of results indicative of bile duct obstruction (raised conjugated
bilirubin, γ-glutamyl transpeptidase and alkaline phosphatase levels). CA19-9 (carbohydrate antigen 19.9)
is a tumor marker that is frequently elevated in pancreatic cancer. Imaging studies, such as computed
tomography (CT scan) and endoscopic ultrasound (EUS) can be used to identify the location and form of
the cancer.

Endoscopic retrograde cholangiopancreatography (choice A) is a diagnostic and therapeutic procedure that


uses endoscopy and fluoroscopy to access the common bile duct and the billiary tree. A endoscopic stent
can be deployed in the common bile duct or to retrieve a stone in order to relieve obstructive jaundice.
Since the source of obstruction is a pancreatic mass and the common bile duct is not dilated, there is no
role for ERCP in this case.

Selective angiography (choice B) is not indicated for pancreatic masses. Angiography is a medical imaging
technique used to visualize blood vessels. This is traditionally done by injecting a radio-opaque contrast
agent into the blood vessel and imaging using X-ray based techniques such as fluoroscopy.

Percutaneous transhepatic cholangiography (PTC) (choice D) is a radiologic technique used to visualize the
anatomy of the biliary tract. A contrast medium is injected into a bile duct in the liver, after which X-rays
are taken. It allows access to the biliary tree in cases where endoscopic retrograde
cholangiopancreatography (ERCP) has been unsuccessful. Since the source of billiary obstruction is outside
the billiary tree, it is not indicated.

Magnetic resonance cholangiopancreatography (choice E) is a medical imaging technique that uses


magnetic resonance imaging to visualise the biliary and pancreatic ducts in a non-invasive manner. This
procedure can be used to determine if gallstones are lodged in any of the ducts surrounding the
gallbladder. In the absence of ductal dilation, this is not the initial study to assess masses in the pancreas.

A 12 year old boy is brought into your office by his mother. She recently noticed a lump on his neck,
which the boy admitted he first felt about nine months ago and seems to be getting larger. Physical
examination reveals a palpable, painless mass of about 2 cm, located just anterior to the trachea in the
midline. A fine needle aspirate of the mass yields only clear, mucoid fluid.

Which of the following is the most likely diagnosis?

Parathyroid cyst
Thyroglossal duct cyst
Follicular adenoma
Nodule of multimodal goiter
Lymph node metastasis of follicular carcinoma
Branchial cleft cysts
Dermoid cyst

Explanation

The correct answer is choice B.

48
A thyroglossal cyst is a fibrous cyst that forms from a persistent thyroglossal duct. It usually presents as a
midline neck lump (in the region of the hyoid bone) that is usually painless, smooth and cystic, if infected
pain can occur. Typically, the cyst will move upwards on protrusion of the tongue, given its attachment to
the embryonic duct. Treatment for a thyroglossal cyst is surgical resection, often requiring concomitant
removal of the midsection of the hyoid bone (Sistrunk procedure), to prevent recurrence. Although
generally benign, the cyst will be removed if the patient exhibits difficulty in breathing or swallowing, or if
the cyst is infected. Even if these symptoms are not present the cyst may be removed to eliminate the
chance of infection or development of a carcinoma, or for cosmetic reasons if there is unsightly protrusion
from the neck.

Parathyroid cysts (choice A) are an uncommon incidental finding at autopsy. They rarely exceed 1 cm.

Follicular adenomas (choice C) are rare at this age. They would likely appear in right or left lobe and be
palpable off the midline.

Multinodular goiters (choice D) produce diffuse, nodular thyroid enlargement, not a midline mass.

Thyroid cancers are rare at this age (except with previous radiation exposure), and lymph node metastases
(choice E) are more common from papillary carcinomas.

Branchial cleft cysts (choice F) are congenital epithelial cysts, which arise on the lateral part of the neck
from a failure of obliteration of the second branchial cleft in embryonic development.

A dermoid cyst (choice G) is a cystic teratoma that contains developmentally mature skin complete with
hair follicles and sweat glands, sometimes clumps of long hair, and often pockets of sebum, blood, fat,
bone, nails, teeth, eyes, cartilage, and thyroid tissue. Because it contains mature tissue, a dermoid cyst is
almost always benign. The rare malignant dermoid cyst usually develops squamous cell carcinoma in
adults; in babies and children it usually develops endodermal sinus tumor.

A 60 year old man presents for followup after being discharged from the hospital with a blood clot in his
left leg a week ago. Upon further questioning, he has had a history of painless jaundice for the past 6
months. He was recently diagnosed with diabetes a month ago. He mentions that his stools have been
pale and his urine has been darker than usual since his last visit. Upon inspection, you see jaundiced man
with scleral icterus.

Which of the following would be the next appropriate management step?

cholecystojejunostomy
percutaneous transhepatic stent
endoscopic stent
endoscopic papillotomy
chemotherapy

Explanation

The correct answer is choice A

Pancreatic cancer is a malignant neoplasm of the pancreas. The prognosis is poor, with fewer than 5% of
those diagnosed still alive five years after diagnosis. Complete remission is still rare. About 95% of
exocrine pancreatic cancers are adenocarcinomas. The remaining 5% include adenosquamous carcinomas,

49
signet ring cell carcinomas, hepatoid carcinomas, colloid carcinomas, undifferentiated carcinomas, and
undifferentiated carcinomas with osteoclast-like giant cells. Exocrine pancreatic tumors are far more
common than pancreatic endocrine tumors, which make up about 1% of total cases

Pancreatic cancer is sometimes called a "silent killer" because early pancreatic cancer often does not cause
symptoms, and the later symptoms are usually nonspecific and varied. Therefore, pancreatic cancer is
often not diagnosed until it is advanced. Common symptoms include:

 Pain in the upper abdomen that typically radiates to the back (seen in carcinoma of the body or
tail of the pancreas)
 Loss of appetite and/or nausea and vomiting
 Significant weight loss
 Painless jaundice (yellow skin/eyes, dark urine) when a cancer of the head of the pancreas (about
60% of cases) obstructs the common bile duct as it runs through the pancreas. This may also
cause pale-colored stool and steatorrhea.
 Trousseau sign, in which blood clots form spontaneously in the portal blood vessels, the deep
veins of the extremities, or the superficial veins anywhere on the body, is sometimes associated
with pancreatic cancer.
 Diabetes mellitus, or elevated blood sugar levels. Many patients with pancreatic cancer develop
diabetes months to even years before they are diagnosed with pancreatic cancer, suggesting new
onset diabetes in an elderly individual may be an early warning sign of pancreatic cancer.

Treatment of pancreatic cancer is mainly surgical and depends on the stage of the cancer. The Whipple
procedure is the most common surgical treatment for cancers involving the head of the pancreas. This
procedure involves removing the pancreatic head and the curve of the duodenum together (pancreato-
duodenectomy), making a bypass for food from stomach to jejunum (gastro-jejunostomy) and attaching a
loop of jejunum to the cystic duct to drain bile (cholecysto-jejunostomy). It can be performed only if the
patient is likely to survive major surgery and if the cancer is localized without invading local structures or
metastasizing. It can, therefore, be performed in only the minority of cases.

percutaneous transhepatic stent ( choice B) may be used to drain bile until a more permanent solution for
the obstruction is performed (e.g. surgery). Additionally, self expanding metal stents can be placed across
malignant biliary strictures to allow palliative drainage. Percutaneous placement of metal stents can be
utilised when therapeutic ERCP has been unsuccessful, anatomy is altered precluding endoscopic access to
the duodenum, or where there has been separation of the segmental biliary drainage of the liver, allowing
more selective placement of metal stents. It is generally accepted that percutanous biliary procedures have
higher complication rates than therapeutic ERCP. Complications encountered include infection, bleeding
and bile leaks.

endoscopic stent (choice C) is a metallic tube used to hold open a structure in the gastrointestinal tract in
order to allow the passage of food, chyme, stool, or other secretions required for digestion. Stents are
inserted by endoscopy, wherein a fibre optic camera is inserted either through the mouth or retrograde
through the colon, in order to reach an area of narrowing.

The vast majority of stents are used to alleviate symptoms caused by cancers of the gastrointestinal tract
that obstruct the interior of the tube-like (or luminal) structures of the bowel — namely the esophagus,
duodenum, common bile duct and colon. Biliary stents are used to palliatively treat tumours of the
pancreas or bile duct that obstruct the common bile duct. They are inserted at the time of ERCP, a
procedure that uses endoscopy and fluoroscopy to access the common bile duct. Stents are not the first
line management of pancreatic head cancers.

endoscopic papillotomy (choice D) is used to relieve stenosis of the sphincter of Oddi, a muscular valve,
that prevents the opening and release of bile or pancreatic fluids into the duodenum in response to food
entering the duodenum. Obstructions are commonly caused by stones or scarring. Obstruction of the valve
can cause pancreatic pain, jaundice, attacks of pancreatitis. Endoscopic papillotomy is not the first line
management of pancreatic head cancers.

chemotherapy ( choice E) is the next appropriate step after pancreatic head resection. After surgery,
adjuvant chemotherapy with gemcitabine has been shown in several large randomized studies to

50
significantly increase the 5-year survival (from approximately 10 to 20%), and should be offered if the
patient is fit after surgery.

An 8 month old present to the hospital with restlessness, refusal to eat and vomiting. There is a history of passage red
currant jelly stool. On examination, he is dehydrated with abdominal distention. A sausage shaped mass is palpated on
the abdomen.

The most likely diagnosis includes which of the following:

Cyclical vomiting syndrome


Intussusception
Colic
Volvulus
Infant botulism

Explanation

The correct answer is Choice B

Intussusception is a medical condition that occurs when a part of the intestine invaginates into the lumen
of the adjoining intestine. The condition typically affects infants between 6 - 8 months of age with an
incidence of 1/500 children. There are two main types:

 Idiopathic intussusception: This is the more common form. It usually starts at the ileo-colic
junction and is seen mostly in infants and toddlers.
 Enteroenteral intussusception: this can be jejunojejunal, jejunoileal or Ileo-ileal. It is commoner in older children
and has been linked with an infective aetiology like Adenovirus. This leads to the enlarged Payers Patches
frequently found at surgery. The enlargement of Peyers Patches are thought to form the axis for the invagination
to occur. There is also an association with conditions like Henoch-Schönlein purpura, Cystic fibrosis and
haematological dyscrasias.

A high index of suspicion is required, as the symptoms are variable. Delay in diagnosis can be costly. The
classic triad of vomiting, abdominal pain and passage of blood per rectum occurs in only a third of patients.
Pain is colicky, severe and intermittent, occurring every 10 – 15 minutes and lasting for about 2 – 3
minutes. During each spasm, the child draws up the legs. Vomiting is an early sign which is initially non-
billous. However, as the intestinal obstruction progresses, vomiting becomes bilious. Passage of blood and
mucus per rectum is often a late feature. This is often referred to as red currant jelly stool, the typical
description of the blood stained stool seen in this condition.

On examination, the child is anxious, ill-looking and usually in pain. Abdominal examination may reveal a
sausage-shaped mass in the right upper quadrant of the abdomen. This may not always be palpable e.g. in
an ileo-ileal intussusception. On rectal examination, the finger is stained with red currant jelly stool.

Urgent serum electrolytes should be requested to assess dehydration. Plain abdominal x-ray will show
signs of intestinal obstruction. Abdominal ultrasound may show the Swiss-roll appearance. Contrast studies
using barium enema may show the Stack of Coins sign. Barium enema has been employed as a means of
reducing the intussusception under radiological control. It is however contraindicated in a suspected case
of bowel gangrene or perforation. Alternatively, hydrostatic reduction with oxygen is used in some centres.
There is a 10% chance of recurrence with this method. Indications for surgical intervention include;

51
 Children under 3 months or over 2 years of age
 Clinical signs of peritonitis
 Gross dehydration
 Failure of medical treatment or recurrence.

Some differential diagnoses include;

Cyclical vomiting syndrome (Choice A) which is characterized by recurrent explosive bouts of vomiting
resulting in time lost from school or work. The disease is commonly seen in children although some cases
have been reported in adults. The aetiology is unknown. Treatment is conservative through provision of
advice on avoiding known triggers, use of anti-emetics, and supportive care during acute attack.

Infant colic (Choice C) is a condition that usually occurs within the first three months of life. It is
characterized by inconsolable episodes of excessive and paroxysmal crying and drawing up of legs. There
is no vomiting. It occurs mostly in the evenings and has no identifiable cause. Colic has been described as
paroxysms of crying, lasting >3hours, occurring >3 days in any week for 3 weeks. Treatment is usually
supportive.

Volvulus (Choice D) refers to the complete twisting of a loop of intestine around it mesenteric origin.
Volvulus neonatorum occurs as a result of failure of normal rotation of the bowel, with the caecum
remaining high in the abdomen and a band across the duodenum which becomes obstructed. Mid-gut
volvulus is a common form which occurs when there is a twisting of the entire mid-gut around the axis of
the superior mesenteric artery. Volvulus is a dangerous condition that may lead to infarction of the entire
mid-gut. Urgent surgical intervention is required to treat the condition.

Infant botulism (Choice E) is an acute neurological condition that results from ingestion of the spores of
Clostridium botulinum spores either from the environment or contaminated honey. The spores then germinate
in the intestines and release toxins, which interferes with the baby’s ability to feed, move, or breathe. It
affects infants between 3 weeks to 6 months of age but children remain susceptible till their first birthday.
Most infants recover from the condition with supportive care with mortality being about 1%.

A 24 year woman presents to bariatric clinic for presurgical evaluation. She has has long standing issues
with her weight and her current BMI is 45 kg/m2. Her medical history is significant for type 2 diabetes and
osteoarthritis. She has difficulties breathing during sleep and is on a CPAP machine to deliver continuous
positive pressure for obstructive sleep apnea.

Which of the following statements regarding surgery for morbid obesity is correct?

Average 30-day mortality ranges from 0.1% to 1.1%


Among major comorbidities that are resolved or improved by surgery, obstructive sleep apnea is
least frequently affected
Compared with an open approach, laparoscopic gastric bypass is associated with lower
incidence of incisional hernias and higher excess weight loss achieved after 3 years
Biliopancreatic diversion is associated with significant postoperative morbidity and mortality and
should be reserved for patients who fail to achieve weight loss with restrictive operations
bariatric surgery has few complications

Explanation

52
The correct answer is choice A

Obesity is a medical condition in which excess body fat has accumulated to the extent that it may have an
adverse effect on health, leading to reduced life expectancy and/or increased health problems. Obesity
increases the likelihood of various diseases, particularly heart disease, type 2 diabetes, breathing
difficulties during sleep, certain types of cancer, and osteoarthritis. Body mass index (BMI), a
measurement which compares weight and height, defines people as overweight (pre-obese) when their
BMI is between 25 kg/m2 and 30 kg/m2, and obese when it is greater than 30 kg/m2. The surgical
literature breaks down "class III" obesity into further categories whose exact values are still disputed.

 Any BMI ≥ 35 or 40 is severe obesity


 A BMI of ≥ 35 or 40–44.9 or 49.9 is morbid obesity
 A BMI of ≥ 45 or 50 is super obese

Bariatric surgery ("weight loss surgery") is the use of surgical intervention in the treatment of obesity. As
every operation may have complications, surgery is only recommended for severely obese people (BMI >
40) who have failed to lose weight following dietary modification and pharmacological treatment. Weight
loss surgery relies on various principles: the two most common approaches are reducing the volume of the
stomach (e.g. by adjustable gastric banding and vertical banded gastroplasty), which produces an earlier
sense of satiation, and reducing the length of bowel that comes into contact with food (gastric bypass
surgery), which directly reduces absorption. Band surgery is reversible, while bowel shortening operations
are not. Some procedures can be performed laparoscopically. Complications from weight loss surgery are
frequent with average 30-day mortality ranging from 0.1% to 1.1%.

Surgery for severe obesity is associated with long-term weight loss and decreased overall mortality. One
study found a weight loss of between 14% and 25% (depending on the type of procedure performed) at 10
years, and a 29% reduction in all cause mortality when compared to standard weight loss measures.

Among major comorbidities that are resolved or improved by surgery, obstructive sleep apnea is least
frequently affected (choice B) is incorrect. Patients that undergo bariatric surgery often experience
symptomatic relief of obstructive sleep apnea with weight loss.

Compared with an open approach, laparoscopic gastric bypass is associated with lower incidence of
incisional hernias and higher excess weight loss achieved after 3 years (choice C) is incorrect. Incisional
hernias and weight loss rates are not significantly different between open versus laproscopic approaches.

Biliopancreatic diversion is associated with significant postoperative morbidity and mortality and should be
reserved for patients who fail to achieve weight loss with restrictive operations (choice D) is not correct.
Biliopancreatic diversion (BPD) involves resecting a portion of the stomach, creating a smaller stomach
(however the patient can eat a free diet as there is no restrictive component). The distal part of the small
intestine is then connected to the pouch, bypassing the duodenum and jejunum.

In around 2% of patients there is severe malabsorption and nutritional deficiency that requires restoration
of the normal absorption. The malabsorptive effect of BPD is so potent that those who undergo the
procedure must take vitamin and dietary minerals above and beyond that of the normal population.
Without these supplements, there is risk of serious deficiency diseases such as anemia and osteoporosis.

bariatric surgery has few complications (choice E) is incorrect. Gallstones are a common complication of
the rapid weight loss following any type of bariatric surgery. Therefore, some surgeons remove
thegallbladder as a preventative measure. Others prefer to prescribe medications to reduce the risk of
post-operative gallstones.

A 62-year-old woman notices blood in the toilet bowel after a bowel movement. She undergoes a
colonoscopy and is found to have a rectal mass in the sigmoid colon. Biopsies of the lesion are taken and
reveal adenocarcinoma.

Which of the following modalities would best assess the local extent of disease in this patient?

53
CT scan of the abdomen and pelvis
Endoscopic ultrasound (EUS)
MRI scan of the abdomen and pelvis
Positron emission tomography (PET) scan of the abdomen and pelvis
Double contrast barium enema

Explanation

The correct answer is choice B

Colorectal cancer, also called colon cancer or large bowel cancer, includes cancerous growths in the colon,
rectum and appendix. It is the fourth most common form of cancer in the United States and the third
leading cause of cancer-related death in the Western world. Colorectal cancers arise from adenomatous
polyps in the colon. These mushroom-shaped growths are usually benign, but some develop into cancer
over time. Localized colon cancer is usually diagnosed through colonoscopy.

Invasive cancers that are confined within the wall of the colon (TNM stages I and II) are curable with
surgery. If untreated, they spread to regional lymph nodes (stage III), where up to 73% are curable by
surgery and chemotherapy. Cancer that metastasizes to distant sites (stage IV) is usually not curable,
although chemotherapy can extend survival, and in rare cases, surgery and chemotherapy together have
seen patients through to a cure. Radiation is used with rectal cancer.

Colon cancer staging is an estimate of the amount of penetration of a particular cancer. It is performed for
diagnostic and research purposes, and to determine the best method of treatment. The systems for staging
colorectal cancers depend on the extent of local invasion, the degree of lymph node involvement and
whether there is distant metastasis.

Definitive staging can only be done after surgery has been performed and pathology reports reviewed. An
exception to this principle would be after a colonoscopic polypectomy of a malignant pedunculated polyp
with minimal invasion. Preoperative staging of rectal cancers may be done with endoscopic ultrasound.
Endoscopic ultrasound is a medical procedure in which endoscopy is combined with ultrasound to obtain
images of the internal organs in the chest and abdomen. It can be used to visualize the wall of these
organs, or to look at adjacent structures. Combined with Doppler imaging, nearby blood vessels can also
be evaluated. Adjunct staging of metastasis include Abdominal Ultrasound, CT, PET Scanning, and other
imaging studies.

CT scan of the abdomen and pelvis (choice A) can be used for adjunct staging. Standard computed axial
tomography is an x-ray method that can be used to determine the degree of spread of cancer.

MRI scan of the abdomen and pelvis (choice C) can be used for adjunct staging but it is not the initial
imaging modality to order to assess for the local extent of disease.

Positron emission tomography (PET) scan of the abdomen and pelvis (choice D) is a 3-dimensional
scanning technology where a radioactive sugar is injected into the patient, the sugar collects in tissues with
high metabolic activity, and an image is formed by measuring the emission of radiation from the sugar.
Because cancer cells often have very high metabolic rates, this can be used to differentiate benign and
malignant tumors. PET is not used for screening and does not (yet) have a place in routine workup of
colorectal cancer cases.

Double contrast barium enema (choice D) can detect a cancer or a precancerous polyp but miss the (less
common) flat polyp. An enema containing barium sulfate is administered, then air is insufflated into the
colon, distending it. The result is a thin layer of barium over the inner lining of the colon which is visible on
X-ray films.

54
A 67 year old man with hepatitis C and liver cirrhosis requires an urgent cholecystectomy. Upon routine
preoperative lab testing, he is found to have abnormal coagulation studies due to hepatic synthetic
dysfunction. He requires an urgent transfusion of fresh frozen plasma to minimize the risk of bleeding due
to surgery.

The optimal timing of this transfusion is which of the following?

the day before surgery


the night before surgery
on call to surgery
intraoperatively
postoperative period

Explanation

The correct answer is choice C

Fresh frozen plasma (FFP) refers to the liquid portion of human blood that has been frozen and preserved
quickly after a blood donation and will be used for blood transfusion. The term "FFP" is often used to mean
any transfused plasma product.

Few specific indications for the use of FFP exist and is optimally delivered right before surgery. These
indications generally are limited to the treatment of deficiencies of coagulation proteins for which specific
factor concentrates are unavailable or undesirable. Indications for the use of FFP include the following:

FFP is efficacious for treatment of deficiencies of factors II, V, VII, IX, X, and XI when specific component
therapy is neither available nor appropriate. Requirements for FFP vary with the specific factor being
replaced. For example, hemostatic levels of factor IX in a patient with severe deficiency are difficult to
achieve with FFP alone, whereas patients with severe factor X deficiency require factor levels of about 10
percent to achieve hemostasis and are easily treated with FFP.

Patients who are anticoagulated with warfarin are deficient in the functional vitamin K dependent
coagulation factors II, VII, IX, and X, as well as proteins C and S. These functional deficiencies can be
reversed by the administration of vitamin K. However, for anticoagulated patients who are actively
bleeding or who require emergency surgery, FFP can be used to achieve immediate hemostasis.

FFP can be used as a source of antithrombin III in patients who are deficient of this inhibitor and are
undergoing surgery or who require heparin for treatment of thrombosis. FFP may be beneficial for the
treatment of thrombotic thrombocytopenic purpura.

FFP is useful in infants with secondary immunodeficiency associated with severe protein-losing enteropathy
and in whom total parenteral nutrition is ineffectual. FFP also can be used as a source of immunoglobulin
for children and adults with humoral immunodeficiency. However, the development of a purified immune
globulin for intravenous use largely has replaced FFP.

The day before surgery (choice A) is not the optimal time to decrease bleeding risk in a patient with known
coagulopathy. The factors need to be thawed and given shortly before surgery.

The night before surgery (choice B) is not the optimal time to decrease bleeding risk in a patient with
known coagulopathy. The factors need to be thawed and given shortly before surgery.

55
Intraoperatively (choice D) is not the optimal time to decrease bleeding risk in a patient with known
coagulopathy.

Postoperative period (choice E) is too late to reverse the coagulopathy and doesn’t address the risk of
intraoperative bleeding.

A 6-year-old child has fever, generalized abdominal pain and tenderness, vomiting, and loose, guaiac-
positive stools. Laboratory evaluation reveals a hematocrit of 29%, leukocyte count of 17.0 x 103/mm 3,
and a platelet count of 62 × 103/mm3.

Which of the following is the most likely diagnosis for this patient:

Appendicitis
Gastroenteritis
Hemolytic uremic syndrome
Intussusception
Henoch Schonlein Purpura

Explanation

The correct answer is choice C.

Hemolytic uremic syndrome, diarrhea-associated type in this case, is a syndrome characterized by a triad
of hemolytic anemia, thrombocytopenia and acute renal failure.It is recognized as the most common cause
of acute renal failure in the pediatric population. Hemolytic-uremic syndrome is classified as diarrhea-
associated (D+ hemolytic-uremic syndrome) and non–diarrhea-associated (D- or atypical hemolytic-uremic
syndrome).

It is caused by gastrointestinal infection by shiga toxin-producing Escherichia coli. The toxin produces
hemorragic colitis causing acute abdomen with occult blood in stools; guaiac-positive stools.

Then the toxin is absorbed to circulation and reaches the kidney causing endothelial cell injury of the
glomerular capillaries with micro thrombi formation inside these capillaries. This leads to diminished
glomerular filtration with hemolysis of the RBCs as they pass through the capillaries. In addition, there is
increased release of a platelet aggregating factor; thrombaxane A2 leading to consumptive
thrombocytopenia.

Answer A is false as appendicitis is not associated with thrombocytopenia, anemia or occult blood in stools.

Answer B is false as gastroenteritis cannot explain the existing thrombocytopenia and the high grade of
anemia.

Answer D is false as intussusception is also not associated with thrombocytopenia or leukocytosis.

Answer E is false. Although Henoch Schonlein Purpura is characterized by thrombocytopenia, abdominal


pain and occult blood in stools, yet the pupura of skin, the most prominent physical finding in this disease,
is missing in this case. Also it is not associated with evident anemia.

56
A 76 year old man is brought to the ER from a nursing home for increasing abdominal pain and distension.
He has been constipated and has not had a bowel movement in over a week. Physical exam reveals a
soft, distended tympanic abdomen with severe tenderness in all four quadrants. Labs are significant for
[HCO3-] of 21 mmol/l and blood pH of 7.31. A CAT scan of the abodmen shows a sigmoid volvulus.

What is the next appropriate step in management?

sigmoidoscopy
endoscopic reduction
fecal disimpaction
barium enema
emergent laparotomy

Explanation

The correct answer is choice E

A volvulus is a bowel obstruction in which a loop of bowel has abnormally twisted on itself. Midgut volvulus
occurs in patients (usually in infants) that are predisposed because of congenital intestinal malrotation.
Segmental volvulus occurs in patients of any age, usually with a predisposition because of abnormal
intestinal contents (e.g. meconium ileus) or adhesions. Volvulus of the cecum, transverse colon, or sigmoid
colon occurs, usually in adults, with only minor predisposing factors such as redundant (excess,
inadequately supported) intestinal tissue and constipation.

Volvulus occurs most frequently in middle-aged and elderly men. Volvulus can also arise as a rare
complication in persons with redundant colon, a normal anatomic variation resulting in extra colonic loops.
Sigmoid volvulus is the most-common form of volvulus of the gastrointestinal tract and is responsible for
8% of all intestinal obstructions. Sigmoid volvulus is particularly common in elderly persons and
constipated patients. Patients experience abdominal pain, distension, and absolute constipation.

Regardless of cause, volvulus causes symptoms by two mechanisms.

 One is bowel obstruction, manifested as abdominal distension and vomiting.


 The other is ischemia (loss of blood flow) to the affected portion of intestine.

Volvulus causes severe pain and progressive injury to the intestinal wall, with accumulation of gas and fluid
in the portion of the bowel obstructed. Ultimately, this can result in necrosis of the affected intestinal wall,
acidosis, and death. Acute volvulus therefore requires immediate surgical intervention to untwist the
affected segment of bowel and possibly resect any unsalvageable portion.

Emergent surgical intervention is indicated for acute gastric volvulus and is still considered a surgical
emergency by many surgeons. With chronic gastric volvulus, surgery is performed to prevent
complications. The principles associated with the treatment of gastric volvulus include decompression,
reduction, and prevention of recurrence, which is best accomplished with surgical therapy.

sigmoidoscopy (choice A) is a minimally invasive medical examination of the large intestine from the
rectum through the last part of the colon through the use of an endoscopic camera. Sigmoidoscopy enables
the physician to look at the inside of the large intestine from the rectum through the last part of the colon,
called the sigmoid to find the cause of diarrhea, abdominal pain, or constipation. However, it is not
necessary after the diagnosis of volvus has been established by CT or plain film abdominal xrays.

endoscopic reduction (choice B) can be attempted in selected patients but is not appropriate in in patients
who appear clinically ill or are found to have vascular compromise during endoscopy. Such treatment can

57
be accomplished by advancing the scope beyond the point of torsion and then rotating it to untwist the
stomach. Nonoperative treatment might be appropriate for older patients that have intestinal malrotation
and are truly asymptomatic. Definitive treatment for sigmoid volvulus remains surgical with resection and
primary anastomosis.

fecal disimpaction (choice C) involves removing a solid, immobile bulk of human feces that can develop in
the rectum as a result of chronic constipation through the use of laxatives or manual disimpaction. While it
may relieve symptomatic relief of constipation, it will not resolve the volvulus.

barium enema (choice D) is used to examine and diagnose problems with the human colon (large
intestine). X-ray pictures are taken while barium sulfate fills the colon via the rectum. Sigmoidoscopy,
rather than barium enema examination, is the procedure of choice if an ileosigmoid knot is suspected. Both
are unnecessary when diagnosis of volvulus has been established by CT or abdominal radiograph.

58

Das könnte Ihnen auch gefallen